Решить 20 задание базового уровня. Подготовка к ЕГЭ по математике (профильный уровень): задания, решения и объяснения

Рассмотрим такого плана задачи. Мы имеем следующие условия:

Общее количество: N

Из А штук хотя бы 1 другого вида, а из В штук хотя бы 1 первого вида

Тогда: (А-1) – минимальное количество первого вида, а (В-1) – второго.

После делаем проверку: (А-1)+(В-1)= N .

ПРИМЕР

В

РЕШЕНИЕ

Итак: всего рыб у нас 35 (окуни и плотвички)

Рассмотрим условия: среди любых 21 рыбы имеется хотя бы одна плотвичка, значит минимум 1 плотвичка есть в данном условии, следовательно (21-1)=20 это минимум окуней. Среди любых 16 рыб - хотя бы один окунь, рассуждая аналогично, (16-1)=15 – это минимум плотвичек. Теперь делаем проверку: 20+15=35, то есть мы получили общее количество рыб, а значит 20 окуней и 15 плотвичек.

ОТВЕТ: 15 плотвичек

    Викторина и количество правильных ответов

Список заданий викторины состоял из А вопросов. За каждый правильный ответ ученик получал а очков, за неправильный ответ с него списывали b очков, а при отсутствии ответа давали 0 очков. Сколько верных ответов дал ученик, набравший N очков, если известно, что по крайней мере один раз он ошибся?

Мы знаем сколько баллов он заработал, знаем цену правильного и неправильного ответа. Исходя из того был дан хотя бы один неправильный ответ, то количество баллов за правильные ответы должны превышать количество штрафных баллов на N баллов. Пусть было дано х правильных ответов и у неправильных, тогда:

а* x = N + b * y

х=( N + b * y )/а

из данного равенства видно, что число в скобках должно быть кратным а. С учетом этого мы можем оценить у(он тоже целое число). При этом надо учесть что количество правильных и не правильных ответов не должно превышать общего числа вопросов.

ПРИМЕР

РЕШЕНИЕ:

вводим обозначения (для удобства) х - правильные, у – неправильные, тогда

5*х=75+11*у

Х=(75+11*у)/5

Так как 75 делится нацело на пять, то и 11*у тоже должно делиться нацело на пять. Поэтому у может принимать значения кратные пяти (5, 10, 15, и т.д.). берем первое значение у=5 тогда х=(75+11*5)/5=26 всего вопросов 26+5=31

У=10 х=(75+11*10)=37 всего ответов 37+10= 47 (больше чем вопросов) не подходит.

Значит всего было: 26 верных и 5 неверных ответов.

ОТВЕТ: 26 верных ответов

    На каком этаже?

Саша пригласил Петю в гости, сказав, что живёт в а подъезде в квартире № N , а этаж сказать забыл. Подойдя к дому, Петя обнаружил, что дом y- этажный. На каком этаже живёт Саша? (На всех этажах число квартир одинаково, номера квартир в доме начинаются с единицы.)

РЕШЕНИЕ

По условию задачи мы знаем номер квартиры, подъезд и количество этажей в доме. Исходя из этих данных, можно сделать оценку количества квартир на этаже. Пусть х - количество квартир на этаже, тогда должно выполняться следующее условие:

А*у*х должно быть больше или равно N

Из этого неравенства оцениваем х

Берем для начала минимальное целое значение х, пусть оно равно с, и делаем проверку: (а-1)*у*с меньше N , а а*у*с больше или равно N .

Выбрав необходимое нам значение х, мы легко можем рассчитать этаж (в): в=(N -( a -1)* c )/ c , причем в – целое число и получая дробное значение, мы берем ближайшее целое(в большую сторону)

ПРИМЕР

РЕШЕНИЕ

Оценим количество квартир на этаже: 7*7*х больше или равно 462, отсюда хбольше или равен 462/(7*7)=9,42 значит минимальный х=10. Делаем проверку: 6*7*10=420 и 7*7*10=490 в итоге мы получили,что квартира по номеру попадает в данный диапозон. Теперь найдем этаж: (462-6*7*10)/10=4,2 значит мальчик живет на пятом этаже.

ОТВЕТ: 5 этаж

    Квартиры, этажи, подъезды

Во всех подъездах дома одинаковое число этажей, и па всех этажах одинаковое число квартир. При этом число этажей в доме больше числа квартир на этаже, число квартир на этаже больше числа подъездов, а число подъездов больше одного. Сколько этажей в доме, если всего в нём Х квартир?

Данный тип задач базируется на следующем условии: если в доме Э – этажей, П – подъездов и К – квартир на этаже, то общее количество квартир в доме должно быть равно Э*П*К=Х. значит нам необходимо Х представить в виде произведения трех чисел не равны 1(по условию задачи). Для этого сделаем разложение числа Х на простые множители. Сделав разложение и учитывая условия задачи мы делаем выборку соответствия чисел и тех условий, которые указаны в задачи.

ПРИМЕР

РЕШЕНИЕ

Представим число 105 в виде произведения простых множителей

105=5*7*3, теперь вернемся к условию задачи: так как число этажей самое большое, то оно равно 7, число квартир на этаже 5, а подъездов – 3.

ОТВЕТ: подъездов - 7, квартир на этаже – 5, подъездов – 3.

    Обмен

В

За а золотых монет получить у серебряные и с медную;

За х серебряных монет получить в золотых и с 1 медную.

У Николая были только серебряные монеты. После обменного пункта серебряных монет у него стало меньше, золотых не появилось, зато появилось С медных. На сколько уменьшилось количество серебряных монет у Николая?

В обменом пунукте есть две схемы обмена:

ПРИМЕР

В обменном пункте можно совершить одну из двух операций:

РЕШЕНИЕ

5 золотых=4 серебрянных+1 медная

10 серебрянных=7 золотых+1 медная

так как не появилось золотых монет, то нам необходима схема обмена без золоты монет. Поэтому количество золотых монет должно быть равным в обоих случаях. Нам надо найти наименьшее общее кратное чисел 5 и 7, и привести наше золото в обоих случаях к нему:

35 золотых=28 серебрянных+7 медных

50 серебрянных=35 золотых+5 медных

в итоге получаем

50 серебрянных=28 серебрянных+12 медных

Мы нашли схему обмена минуя золотые монеты, теперь нам надо,зная количество медных монет, найти сколько раз такая операция выполнялась

N =60/12=5

В итоге получаем

250 серебрянных=140 серебрянных+60 медных

Подставив, и получив конечный обмен мы найдем какое количество серебра было поменяно. Значит - количество уменьшилось на 250-140=110

ОТВЕТ на 110 монет

6. ГЛОБУС

На поверхности глобуса маркером проведены х параллелей и у меридиана. На сколько частей проведенные линии разделили поверхность глобуса? (меридиан - это дуга окружности, соединяющая Северный и Южный полюсы, а параллель - это граница сечения глобуса плоскостью, параллельной плоскости экватора).

РЕШЕНИЕ:

Так как параллель эо граница сечения глобуса плоскостью, то одна разобьет глобус на 2 части, две на три части, х на х+1 частей

Меридиан же это дуга окружности(точнее полуокружность) и у меридиан разбивают поверхность на у частей следовательно всего получиться (х+1)*у частей.

ПРИМЕР

Проведя аналогичные рассуждения мы получим:

(30+1)*24=744 (части)

ОТВЕТ: на 744 части

7. РАСПИЛЫ

На палке отмечены поперечные линии красного, жёлтого и зелёного цвета. Если распилить палку по красным линиям, получится А кусков, если по жёлтым - В кусков, а если по зелёным - С кусков. Сколько кусков получится, если распилить палку по линиям всех трёх цветов?

РЕШЕНИЕ

Для решения учтем, что количество кусков на 1 больше количества распилов. Теперь необходимо найти сколько линий отмечено на палке. Получаем красных (А-1), желтых – (В-1), зеленных – (С-1). Найдя количество линий каждого цвета и просуммировав их получим общее количество линий: (А-1)+(В-1)+(С-1). Прибавляем к полученному числу единицу (так как количество кусков на один больше количества распилов) получаем количество кусков, если пилить по всем линиям.

ПРИМЕР

На палке отмечены поперечные линии красного, жёлтого и зелёного цвета. Если распилить палку по красным линиям, получится 7 кусков, если по жёлтым - 13 кусков, а если по зелёным - 5 кусков. Сколько кусков получится, если распилить палку по линиям всех трёх цветов?

РЕШЕНИЕ

Находим количество линий

Красных: 7-1=6

Желтых: 13-1=12

Зеленых: 5-1=4

Общее количество линий: 6+12+4=22

Тогда количество кусков: 22+1=23

ОТВЕТ: 23 куска

8. СТОЛБЦЫ И СТРОКИ

В каждую клетку таблицы поставили по натуральному числу так, что сумма всех чисел в первом столбце равна С1, во втором - С2, в третьем - С3, а сумма чисел в каждой строке больше У1, но меньше У2. Сколько всего строк в таблице?

РЕШЕНИЕ

Так как числа в ячейках таблицы не меняются, то сумма всех чисел таблицы равна: С=С1+С2+С3.

Теперь обратим внимание на то, что таблица состоит из натуральных чисел, а значит сумма чисел по строкам должны быть целыми числами и находиться в пределах от (У1+1) до (У2-1)(так как сумма строк ограниченна строго). Теперь мы можем оценить количесво строк:

С/(У1+1) – максимальное количество

С/(У2-1) – минимальное количество

ПРИМЕР

В таблице три столбца и несколько строк. В

РЕШЕНИЕ

Найдем сумму таблицы

С=85+77+71=233

Определим границы суммы строк

12+1=13 – минимальная

15-1=14 – максимальная

Оценим количество строк в таблице

233/13=17,92 максимальное

233/14=16,64 минимальное

В этих пределах заключено только одно целое число – 17

ОТВЕТ: 17

9. ЗАПРАВКА НА КОЛЬЦЕВОЙ

и Г. Расстояние между А и Б - 35 км, между А и В - 20 км, между В и Г - 20 км, между Г и А и В.

РЕШЕНИЕ

Внимательно прочитав задачу, мы заметим, что практически окружность разбита на три дуги АВ, ВГ и АГ. На основании этого мы найдем длину всей окружности(кольцевой). Для данной задачи она равна 20+20+30=70 (км).

Теперь расставив все точки на окружности и подписав длины соответствующих дуг, легко определить искомое расстояние. В данной задаче БВ=АБ-АВ, то есть БВ=35-20=15

ОТВЕТ: 15 км

10. КОМБИНАЦИИ

РЕШЕНИЕ

Для решения данного типа задач следует вспомнить что такое факториал

Факториалом числа N ! называется произведение последовательных чисел от 1 до N , то есть 4!=1*2*3*4.

Теперь вернемся к задаче. Найдем общее количество кубиков: 3+1+1=5. Так как одного цвета у нас три кубика, то общее количество кубиков можно найти по формуле 5!/3! Получим (5*4*3*2*1)/(1*2*3)=5*4=20

ОТВЕТ: 20 способов расстановки

11 . КОЛОДЦЫ

Хозяин договорился с рабочими, что они выкопают ему колодец на следующих условиях: за первый метр он заплатит им Х рублей, а за каждый следующий метр - на У рублей больше, чем за предыдущий. Сколько рублей хозяин должен будет заплатить рабочим, если они выкопают колодец глубиной N метров?

РЕШЕНИЕ:

Так как хозяин увеличивает цену за каждый метр, то за второй он заплатит (Х+У), за третий – (Х+2У), за четвертый (Х+3У) и т.д. Не сложно увидеть, что данная система оплаты напомин6ает арифметическую прогрессия, где а1=Х, d = Y , n = N . Тогда

Оплата за работу есть ничто иное как сумма данной прогрессии:

S = ( (2a +d(n-1))/2)·n

ПРИМЕР:

РЕШЕНИЕ

Исходя из выше сказанного получаем a 1=4200

d=1300

n=11

подставляя эти данные в нашу формулу получаем

S=((2*4200+1300(11-1)/2)*11=((8400+13000)/2)*11=10700*11=117700

ОТВЕТ: 117700

12 . СТОЛБЫ И ПРОВОДА

Х столбов, соеденны между собой проводами, так что от каждого отходит ровно У проводов. Сколько всего проводов натянуто между столбами?

РЕШЕНИЕ

Найдем сколько промежутков между столбами. Между двумя один промежуток, между тремя – два, между четырьмя – 3, между Х – (Х-1).

На каждом промежутке У проводов, тогда (Х-1)*У это всего проводов между столбами.

ПРИМЕР

Десять столбов соеденны между собой проводами, так что от каждого отходит ровно 6 проводов. Сколько всего проводов натянуто между столбами?

РЕШЕНИЕ

Возвращаясь к предыдущим обозначениям получаем:

Х=9 У=6

Тогда получаем (9-1)*6=8*6=48

ОТВЕТ: 48

13. ПИЛИМ ДОСКИ И БРЕВНА

Было несколько брёвен. Сделали Х распилов и получилось У чурбачков. Сколько брёвен распилили?

РЕШЕНИЕ

При решении сделаем одно замечание: некоторые задачи не всегда имеют математическое решение.

Теперь к задаче. При решении надо учесть, что бревен больше чем одно и при распили каждого бревна получается =1 кусок.

Данный вид задачи решать удобнее методом подбора:

Пусть будет два бревна тогда кусков получиться 13+2=15

Возьмем три получим 13+3=16

И тут можно увидеть зависимость, что количество распилов и кусков увеличивается одинаково, то есть количество бревен которые надо распилить равно У-Х

ПРИМЕР

Было несколько брёвен. Сделали 13 распилов и получилось 20 чубачков. Сколько брёвен распилили?

РЕШЕНИЕ

Вернувшись к нашим рассуждениям мы можем подбирать, или можно просто 20-13=7 значит всего 7 бревен

Ответ 7

14 . ВЫПАВШИЕ СТРАНИЦЫ

Из книги выпало подряд несколько страниц. Первая из выпавших страниц имеет номер Х, а номер последней записывается такими же цифрами в каком-то другом порядке. Сколько страниц выпало из книги?

РЕШЕНИЕ

Нумерация страниц, которые выпали, начинаются с нечетного числа и должны оканчиваться четным числом. Поэтому, мы, зная.что номер последней выпавшей записывается теми же цифрами, что первая выпавшая знаем ее последнюю цифру. Путем перестановок оставшихся цифр и, учитывая, что нумерация страницы должна быть больше, чем первая выпавшая, получаем ее номер. Зная номера страниц, можно посчитать сколько их выпало, при этом учтем что страница Х тоже выпала. Значит из получившегося номера мы должны вычисть число (Х-1)

ПРИМЕР

Из книги выпало подряд несколько страниц. Первая из выпавших страниц имеет номер 387, а номер последней записывается такими же цифрами в каком-то другом порядке. Сколько страниц выпало из книги?

РЕШЕНИЕ

Опираясь, на наши рассуждения получаем, что номер последней выпавшей страницы должен оканчиваться на цифру 8. Значит у нас всего два варианта чисел это 378 и 738. 378 нам не подходит так как оно меньше номера первой выпавшей страницы значит последняя выпавшая это 738.

738-(387-1)=352

ОТВЕТ: 352

Следует добавить следующее: иногда просят указать количество листов, тогда следует количество страниц разделить пополам.

15. ИТОГОВАЯ ОЦЕНКА

В конце четверти Вовочка выписал подряд в строчку свои текущие отметки по пению и поставил между некоторых из них знак умножения. Произведения получившихся чисел оказалось равны Х. Какая отметка выходит у Вовочки в четверти по пению?

РЕШЕНИЕ

При решении данного типа задач необходимо учитывать, что его оценки должны быть 2,3,4 и 5. Поэтому нам необходимо разложить число Х на множители 2,3,4 и 5. Причем остаток от разложения тоже должен состоять из этих чисел.

ПРИМЕР1

В конце четверти Вовочка выписал подряд в строчку свои текущие отметки по пению и поставил между некоторых из них знак умножения. Произведения получившихся чисел оказалось равны 2007. Какая отметка выходит у Вовочки в четверти по пению?

РЕШЕНИЕ

Разложим число 2007 на множители

Получим 2007=3*3*223

Значит его отметки: 3 3 2 2 3 теперь найдем среднее арифметическое его оценок для данного набора это 2,6 следовательно его оценка три (больше чем 2,5)

ОТВЕТ 3

ПРИМЕР 2

В конце четверти Вовочка выписал подряд все свои отметки по одному из предметов, их оказалось 5, и поставил между некоторыми из них знаки умножения. Произведение получившихся чисел оказалась равным 690. Какая отметка выходит у Вовочки в четверти по этому предмету, если учитель ставит только отметки 2, 3, 4 и 5 и итоговая отметка в четверти является средним арифметическим всех текущих отметок, округленная по правилам округления? (Например: 2,4 округляется до двух; 3,5 – до 4; а 4,8 – до 5.)

РЕШЕНИЕ

690 разложим на множители так что бы остаток от разложения состоял из цифр 2 3 4 5

690=3*5*2*23

Следовательно его оценки: 3 5 2 2 3

Найдем среднее арифметическое этих чисел: (3+5+2+2+3)/5=3

Это и будет его оценкой

ОТВЕТ: 3

16 . МЕНЮ

В меню ре­сто­ра­на име­ет­ся Х видов са­ла­тов, У вида пер­вых блюд, А видов вто­рых блюд и В вида де­сер­та. Сколь­ко ва­ри­ан­тов обеда из са­ла­та, пер­во­го, вто­ро­го и де­сер­та могут вы­брать по­се­ти­те­ли этого ре­сто­ра­на?

РЕШЕНИЕ

При решении немного урежем меню: пусть есть только салат и первое тогда вариантов становиться (Х*У). Теперь добавим второе блюдо количество вариантов возрастает в А раз и становиться (Х*У*А). ну а теперь добавим десерт. Количество вариантов возрастет в В раз

Теперь мы получаем окончательный ответ:

N= Х*У*А*В

ПРИМЕР

РЕШЕНИЕ
Опираясь на выше изложенное получаем:

N=6*3*5*4=360

ОТВЕТ: 360

17 . ДЕЛИМ БЕЗ ОСТАТКА

В данном разделе рассмотрим задачи на конкретном примере, для большей наглядности

Так как у нас произведение последовательно идущих чисел и их больше чем 7, то хотя бы одно должно делиться на 7. Значит мы имеем произведение, один из множителей которого делиться на 7, следовательно и все произведение тоже делиться на семь, а значит остаток от деления будет равен нулю, или для второй задачи количество множителей должно равняться делителю.

18.ТУРИСТЫ

Данный тип задач тоже рассмотрим на конкретном примере.

Для начала определим, что нам необходимо найти: времямаршрута=подъем+отдых+спуск

Отдых мы знаем, теперь надо найти время подъема и спуска

Читая задачу, мы видим что в обоих случая (подъем и спуск) время зависит как арифметическая прогрессия, но мы еще не знаем на какую высоту было восхождение, хотя ее нетрудно найти:

H =(95-50)15+1=4

Мы нашли высоту подъема, теперь найдем время подъема как сумму арифметической прогрессии: Тподьема= ((2*50+15*(4-1))*4)/2=290 минут

Аналогично находим, учитывая что теперь разность прогрессии равна -10. Получаем Тспуска=((2*60-10(4-1))*4)/2= 180 минут.

Зная все составляющие можно посчитать общее время маршрута:

Тмаршрута=290+180+10=480 минут или переводя в часы(делим на 60) получим 8 часов.

ОТВЕТ: 8 часов

19.ПРЯМОУГОЛЬНИКИ

На прямоугольники встречается два типа задач: на периметры и на площади

Для решения такого плана задач, нетрудно доказать, что при разбитии любого прямоугольника двумя прямолинейными разрезами, мы получим четыре прямоугольника для которых всегда будут выполняться следующие соотношения:

Р1+Р2=Р3+Р4

S1*S2=S3*S4,

где Р периметр , S - площадь

Основываясь на этих соотношениях, мы легко можем решить следующие задачи

19.1.Периметры

РЕШЕНИЕ

Опираясь на выше сказанное получаем

24+16=28+Х

Х=(24+16)-28=12

ОТВЕТ: 12

19.2 ПЛОЩАДИ

Прямоугольник разбит на четыре маленьких прямоугольника двумя прямолинейными разрезами. Площади трёх из них начиная с левого верхнего и далее по часовой стрелке равны 18, 12 и 20. Найдите площадь четвёртого прямоугольника.

РЕШЕНИЕ

Для полученных прямоугольников должно выполняться:

18*20=12*Х

Тогда Х=(18*20)/12=30

ОТВЕТ: 30

20. ТУДА-СЮДА

Улит­ка за день за­пол­за­ет вверх по де­ре­ву на А м, а за ночь спол­за­ет на В м. Вы­со­та де­ре­ва С м. За сколь­ко дней улит­ка впер­вые до­ползёт до вер­ши­ны де­ре­ва?

РЕШЕНИЕ

За одни сутки улитка может подняться на высоту (А-В) метров. Так как она за один день может подняться на высоту А, то до последнего подъема ей необходимо преодолеть высоту (С-А). Исходя из этого, получаем что она будет подниматься (С-А)\(А-В)+1 (единицу прибавляем так как она за один день поднимается на высоту А).

ПРИМЕР

РЕШЕНИЕ

Возвращаясь к нашим рассуждениям получаем

(10-4)/(4-3)+1=7

ОТВЕТ за 7 дней

Следует отметить что таким способ можно решать задачи на наполнение чего либо, когда поступает что-то и что-то вытекает.

21. ПРЫЖКИ ПО ПРЯМОЙ

Кузнечик прыгает вдоль координатной прямой в любом направлении на единичный отрезок за прыжок. Сколько существует различных точек на координатной прямой, в которых кузнечик может оказаться, сделав Х прыжков, начиная прыгать из начала координат?

РЕШЕНИЕ

Предположим, что кузнечик делает все прыжки в одну сторону, тогда он попадет в точку с координатой Х. Теперь он прыгает вперед на (Х-1) прыжков и один обратно: попадает в точку с координатой (Х-2). Рассматривая таким способом все его прыжки можно заметить, что он будет находиться в точках с координатами Х, (Х-2),(Х-4) и т.д. Данная зависимость является не чем иным как арифметической прогрессией с разностью d =-2 и а1=Х, а an =- X . Тогда количество членов этой прогрессии и есть количество точек в которых он может оказаться. Найдем их

an=a1+d(n-1)

X=X+d(n-1)

2X=-2(n-1)

n=X+1

ПРИМЕР

РЕШЕНИЕ

Основываясь на выше приведенных выводах получаем

10+1=11

ОТВЕТ 11 точек

ЗАДАЧИ ДЛЯ САМОСТОЯТЕЛЬНОГО РЕШЕНИЯ:

1. Каж­дую се­кун­ду бак­те­рия де­лит­ся на две новые бак­те­рии. Из­вест­но, что весь объём од­но­го ста­ка­на бак­те­рии за­пол­ня­ют за 1 час. За сколь­ко се­кунд ста­кан будет за­пол­нен бак­те­ри­я­ми на­по­ло­ви­ну?

2. На палке от­ме­че­ны по­пе­реч­ные линии крас­но­го, жёлтого и зелёного цвета. Если рас­пи­лить палку по крас­ным ли­ни­ям, по­лу­чит­ся 15 кус­ков, если по жёлтым - 5 кус­ков, а если по зелёным - 7 кус­ков. Сколь­ко кус­ков по­лу­чит­ся, если рас­пи­лить палку по ли­ни­ям всех трёх цве­тов?

3. Куз­не­чик пры­га­ет вдоль ко­ор­ди­нат­ной пря­мой в любом на­прав­ле­нии на еди­нич­ный от­ре­зок за один пры­жок. Куз­не­чик на­чи­на­ет пры­гать из на­ча­ла ко­ор­ди­нат. Сколь­ко су­ще­ству­ет раз­лич­ных точек на ко­ор­ди­нат­ной пря­мой, в ко­то­рых куз­не­чик может ока­зать­ся, сде­лав ровно 11 прыж­ков?

4. В кор­зи­не лежит 40 гри­бов: ры­жи­ки и груз­ди. Из­вест­но, что среди любых 17 гри­бов име­ет­ся хотя бы один рыжик, а среди любых 25 гри­бов хотя бы один груздь. Сколь­ко ры­жи­ков в кор­зи­не?

5. Саша при­гла­сил Петю в гости, ска­зав, что живёт в седь­мом подъ­ез­де в квар­ти­ре № 462, а этаж ска­зать забыл. По­дой­дя к дому, Петя об­на­ру­жил, что дом се­ми­этаж­ный. На каком этаже живёт Саша? (На всех эта­жах число квар­тир оди­на­ко­во, но­ме­ра квар­тир в доме на­чи­на­ют­ся с еди­ни­цы.)

6. Саша при­гла­сил Петю в гости, ска­зав, что живёт в вось­мом подъ­ез­де в квар­ти­ре № 468, а этаж ска­зать забыл. По­дой­дя к дому, Петя об­на­ру­жил, что дом две­на­дца­ти­этаж­ный. На каком этаже живёт Саша? (На всех эта­жах число квар­тир оди­на­ко­во, но­ме­ра квар­тир в доме на­чи­на­ют­ся с еди­ни­цы.)

7. Саша при­гла­сил Петю в гости, ска­зав, что живёт в две­на­дца­том подъ­ез­де в квар­ти­ре № 465, а этаж ска­зать забыл. По­дой­дя к дому, Петя об­на­ру­жил, что дом пя­ти­этаж­ный. На каком этаже живёт Саша? (На всех эта­жах число квар­тир оди­на­ко­во, но­ме­ра квар­тир в доме на­чи­на­ют­ся с еди­ни­цы.)

8. Саша при­гла­сил Петю в гости, ска­зав, что живёт в де­ся­том подъ­ез­де в квар­ти­ре № 333, а этаж ска­зать забыл. По­дой­дя к дому, Петя об­на­ру­жил, что дом де­вя­ти­этаж­ный. На каком этаже живёт Саша? (На всех эта­жах число квар­тир оди­на­ко­во, но­ме­ра квар­тир в доме на­чи­на­ют­ся с еди­ни­цы.)

9. Тре­нер по­со­ве­то­вал Ан­дрею в пер­вый день за­ня­тий про­ве­сти на бе­го­вой до­рож­ке 15 минут, а на каж­дом сле­ду­ю­щем за­ня­тии уве­ли­чи­вать время, про­ведённое на бе­го­вой до­рож­ке, на 7 минут. За сколь­ко за­ня­тий Ан­дрей про­ведёт на бе­го­вой до­рож­ке в общей слож­но­сти 2 часа 25 минут, если будет сле­до­вать со­ве­там тре­не­ра?

10. Врач про­пи­сал па­ци­ен­ту при­ни­мать ле­кар­ство по такой схеме: в пер­вый день он дол­жен при­нять 3 капли, а в каж­дый сле­ду­ю­щий день - на 3 капли боль­ше, чем в преды­ду­щий. При­няв 30 ка­пель, он ещё 3 дня пьёт по 30 ка­пель ле­кар­ства, а потом еже­днев­но умень­ша­ет приём на 3 капли. Сколь­ко пу­зырь­ков ле­кар­ства нужно ку­пить па­ци­ен­ту на весь курс приёма, если в каж­дом со­дер­жит­ся 20 мл ле­кар­ства (что со­став­ля­ет 250 ка­пель)?

11. Врач про­пи­сал па­ци­ен­ту при­ни­мать ле­кар­ство по такой схеме: в пер­вый день он дол­жен при­нять 20 ка­пель, а в каж­дый сле­ду­ю­щий день - на 3 капли боль­ше, чем в преды­ду­щий. После 15 дней приёма па­ци­ент де­ла­ет пе­ре­рыв в 3 дня и про­дол­жа­ет при­ни­мать ле­кар­ство по об­рат­ной схеме: в 19-й день он при­ни­ма­ет столь­ко же ка­пель, сколь­ко и в 15-й день, а затем еже­днев­но умень­ша­ет дозу на 3 капли, пока до­зи­ров­ка не ста­нет мень­ше 3 ка­пель в день. Сколь­ко пу­зырь­ков ле­кар­ства нужно ку­пить па­ци­ен­ту на весь курс приёма, если в каж­дом со­дер­жит­ся 200 ка­пель?

12. Про­из­ве­де­ние де­ся­ти иду­щих под­ряд чисел раз­де­ли­ли на 7. Чему может быть равен оста­ток?

13. Сколь­ки­ми спо­со­ба­ми можно по­ста­вить в ряд два оди­на­ко­вых крас­ных ку­би­ка, три оди­на­ко­вых зелёных ку­би­ка и один синий кубик?

14. В бак объёмом 38 лит­ров каж­дый час, на­чи­ная с 12 часов, на­ли­ва­ют пол­ное ведро воды объёмом 8 лит­ров. Но в днище бака есть не­боль­шая щель, и из неё за час вы­те­ка­ет 3 литра. В какой мо­мент вре­ме­ни (в часах) бак будет за­пол­нен пол­но­стью.

15. Какое наи­мень­шее число иду­щих под­ряд чисел нужно взять, чтобы их про­из­ве­де­ние де­ли­лось на 7?

16. В ре­зуль­та­те па­вод­ка кот­ло­ван за­пол­нил­ся водой до уров­ня 2 метра. Стро­и­тель­ная помпа не­пре­рыв­но от­ка­чи­ва­ет воду, по­ни­жая её уро­вень на 20 см в час. Под­поч­вен­ные воды, на­о­бо­рот, по­вы­ша­ют уро­вень воды в кот­ло­ва­не на 5 см в час. За сколь­ко часов ра­бо­ты помпы уро­вень воды в кот­ло­ва­не опу­стит­ся до 80 см?

17. В меню ре­сто­ра­на име­ет­ся 6 видов са­ла­тов, 3 вида пер­вых блюд, 5 видов вто­рых блюд и 4 вида де­сер­та. Сколь­ко ва­ри­ан­тов обеда из са­ла­та, пер­во­го, вто­ро­го и де­сер­та могут вы­брать по­се­ти­те­ли этого ре­сто­ра­на?

18. Неф­тя­ная ком­па­ния бурит сква­жи­ну для до­бы­чи нефти, ко­то­рая за­ле­га­ет, по дан­ным гео­ло­го­раз­вед­ки, на глу­би­не 3 км. В те­че­ние ра­бо­че­го дня бу­риль­щи­ки про­хо­дят 300 мет­ров в глу­би­ну, но за ночь сква­жи­на вновь «за­или­ва­ет­ся», то есть за­пол­ня­ет­ся грун­том на 30 мет­ров. За сколь­ко ра­бо­чих дней неф­тя­ни­ки про­бу­рят сква­жи­ну до глу­би­ны за­ле­га­ния нефти?

19. Какое наи­мень­шее число иду­щих под­ряд чисел нужно взять, чтобы их про­из­ве­де­ние де­ли­лось на 9?

20.

за 2 зо­ло­тых мо­не­ты по­лу­чить 3 се­реб­ря­ных и одну мед­ную;

за 5 се­реб­ря­ных монет по­лу­чить 3 зо­ло­тых и одну мед­ную.

21. На по­верх­но­сти гло­бу­са фло­ма­сте­ром про­ве­де­ны 12 па­рал­ле­лей и 22 ме­ри­ди­а­на. На сколь­ко ча­стей про­ведённые линии раз­де­ли­ли по­верх­ность гло­бу­са?

Ме­ри­ди­ан - это дуга окруж­но­сти, со­еди­ня­ю­щая Се­вер­ный и Южный по­лю­сы. Па­рал­лель - это окруж­ность, ле­жа­щая в плос­ко­сти, па­рал­лель­ной плос­ко­сти эк­ва­то­ра.

22. В кор­зи­не лежит 50 гри­бов: ры­жи­ки и груз­ди. Из­вест­но, что среди любых 28 гри­бов име­ет­ся хотя бы один рыжик, а среди любых 24 гри­бов хотя бы один груздь. Сколь­ко груз­дей в кор­зи­не?

23. Груп­па ту­ри­стов пре­одо­ле­ла гор­ный пе­ре­вал. Пер­вый ки­ло­метр подъёма они пре­одо­ле­ли за 50 минут, а каж­дый сле­ду­ю­щий ки­ло­метр про­хо­ди­ли на 15 минут доль­ше преды­ду­ще­го. По­след­ний ки­ло­метр перед вер­ши­ной был прой­ден за 95 минут. После де­ся­ти­ми­нут­но­го от­ды­ха на вер­ши­не ту­ри­сты на­ча­ли спуск, ко­то­рый был более по­ло­гим. Пер­вый ки­ло­метр после вер­ши­ны был прой­ден за час, а каж­дый сле­ду­ю­щий на 10 минут быст­рее преды­ду­ще­го. Сколь­ко часов груп­па за­тра­ти­ла на весь марш­рут, если по­след­ний ки­ло­метр спус­ка был прой­ден за 10 минут.

24. На коль­це­вой до­ро­ге рас­по­ло­же­ны че­ты­ре бен­зо­ко­лон­ки: A, B, C и D. Рас­сто­я­ние между A и B - 35 км, между A и C - 20 км, между C и D - 20 км, между D и A - 30 км (все рас­сто­я­ния из­ме­ря­ют­ся вдоль коль­це­вой до­ро­ги в крат­чай­шую сто­ро­ну). Най­ди­те рас­сто­я­ние между B и C. Ответ дайте в ки­ло­мет­рах.

25. На коль­це­вой до­ро­ге рас­по­ло­же­ны че­ты­ре бен­зо­ко­лон­ки: A, B, C и D. Рас­сто­я­ние между A и B - 50 км, между A и C - 40 км, между C и D - 25 км, между D и A - 35 км (все рас­сто­я­ния из­ме­ря­ют­ся вдоль коль­це­вой до­ро­ги в крат­чай­шую сто­ро­ну). Най­ди­те рас­сто­я­ние между B и C.

26. В клас­се учит­ся 25 уча­щих­ся. Не­сколь­ко из них хо­ди­ли в кино, 18 че­ло­век хо­ди­ли в театр, причём и в кино, и в театр хо­ди­ли 12 че­ло­век. Из­вест­но, что трое не хо­ди­ли ни в кино, ни в театр. Сколь­ко че­ло­век из клас­са хо­ди­ли в кино?

27. По эм­пи­ри­че­ско­му за­ко­ну Мура сред­нее число тран­зи­сто­ров на мик­ро­схе­мах каж­дый год удва­и­ва­ет­ся. Из­вест­но, что в 2005 году сред­нее число тран­зи­сто­ров на мик­ро­схе­ме рав­ня­лось 520 млн. Опре­де­ли­те, сколь­ко в сред­нем мил­ли­о­нов тран­зи­сто­ров было на мик­ро­схе­ме в 2003 году.

28. В пер­вом ряду ки­но­за­ла 24 места, а в каж­дом сле­ду­ю­щем на 2 боль­ше, чем в преды­ду­щем. Сколь­ко мест в вось­мом ряду?

29. На палке от­ме­че­ны по­пе­реч­ные линии крас­но­го, жёлтого и зелёного цвета. Если рас­пи­лить палку по крас­ным ли­ни­ям, то по­лу­чит­ся 5 кус­ков, если по жёлтым - 7 кус­ков, а если по зелёным - 11 кус­ков. Сколь­ко кус­ков по­лу­чит­ся, если рас­пи­лить палку по ли­ни­ям всех трёх цве­тов?

30. В ма­га­зи­не бы­то­вой тех­ни­ки объём про­даж хо­ло­диль­ни­ков носит се­зон­ный ха­рак­тер. В ян­ва­ре было про­да­но 10 хо­ло­диль­ни­ков, и в три по­сле­ду­ю­щих ме­ся­ца про­да­ва­ли по 10 хо­ло­диль­ни­ков. С мая про­да­жи уве­ли­чи­ва­лись на 15 еди­ниц по срав­не­нию с преды­ду­щим ме­ся­цем. С сен­тяб­ря объём про­даж начал умень­шать­ся на 15 хо­ло­диль­ни­ков каж­дый месяц от­но­си­тель­но преды­ду­ще­го ме­ся­ца. Сколь­ко хо­ло­диль­ни­ков про­дал ма­га­зин за год?

31. В об­мен­ном пунк­те можно со­вер­шить одну из двух опе­ра­ций:

1) за 3 зо­ло­тых мо­не­ты по­лу­чить 4 се­реб­ря­ных и одну мед­ную;

2) за 6 се­реб­ря­ных монет по­лу­чить 4 зо­ло­тых и одну мед­ную.

У Ни­ко­лы были толь­ко се­реб­ря­ные мо­не­ты. После по­се­ще­ний об­мен­но­го пунк­та се­реб­ря­ных монет у него стало мень­ше, зо­ло­тых не по­яви­лось, зато по­яви­лось 35 мед­ных. На сколь­ко умень­ши­лось ко­ли­че­ство се­реб­ря­ных монет у Ни­ко­лы?

32. Саша при­гла­сил Петю в гости, ска­зав, что живёт в седь­мом подъ­ез­де в квар­ти­ре № 462, а этаж ска­зать забыл. По­дой­дя к дому, Петя об­на­ру­жил, что дом се­ми­этаж­ный. На каком этаже живёт Саша? (На каж­дом этаже число квар­тир оди­на­ко­во, но­ме­ра квар­тир в доме на­чи­на­ют­ся с еди­ни­цы.)

33. Во всех подъ­ез­дах дома оди­на­ко­вое число эта­жей, а на каж­дом этаже оди­на­ко­вое число квар­тир. При этом число эта­жей в доме боль­ше числа квар­тир на этаже, число квар­тир на этаже боль­ше числа подъ­ез­дов, а число подъ­ез­дов боль­ше од­но­го. Сколь­ко эта­жей в доме, если всего в нём 110 квар­тир?

34. Куз­не­чик пры­га­ет вдоль ко­ор­ди­нат­ной пря­мой в любом на­прав­ле­нии на еди­нич­ный от­ре­зок за пры­жок. Сколь­ко су­ще­ству­ет раз­лич­ных точек на ко­ор­ди­нат­ной пря­мой, в ко­то­рых куз­не­чик может ока­зать­ся, сде­лав ровно 6 прыж­ков, на­чи­ная пры­гать из на­ча­ла ко­ор­ди­нат?

35. В кор­зи­не лежат 40 гри­бов: ры­жи­ки и груз­ди. Из­вест­но, что среди любых 17 гри­бов име­ет­ся хотя бы один рыжик, а среди любых 25 гри­бов хотя бы один груздь. Сколь­ко ры­жи­ков в кор­зи­не?

36. В кор­зи­не лежат 25 гри­бов: ры­жи­ки и груз­ди. Из­вест­но, что среди любых 11 гри­бов име­ет­ся хотя бы один рыжик, а среди любых 16 гри­бов хотя бы один груздь. Сколь­ко ры­жи­ков в кор­зи­не?

37. В кор­зи­не лежат 30 гри­бов: ры­жи­ки и груз­ди. Из­вест­но, что среди любых 12 гри­бов име­ет­ся хотя бы один рыжик, а среди любых 20 гри­бов хотя бы один груздь. Сколь­ко ры­жи­ков в кор­зи­не?

38. На гло­бу­се фло­ма­сте­ром про­ве­де­ны 17 па­рал­ле­лей (вклю­чая эк­ва­тор) и 24 ме­ри­ди­а­на. На сколь­ко ча­стей про­ведённые линии раз­де­ля­ют по­верх­ность гло­бу­са?

39. Улит­ка за день за­пол­за­ет вверх по де­ре­ву на 4 м, а за ночь спол­за­ет на 3 м. Вы­со­та де­ре­ва 10 м. За сколь­ко дней улит­ка впер­вые до­ползёт до вер­ши­ны де­ре­ва?

40. Улит­ка за день за­пол­за­ет вверх по де­ре­ву на 4 м, а за ночь спол­за­ет на 1 м. Вы­со­та де­ре­ва 13 м. За сколь­ко дней улит­ка впер­вые до­ползёт до вер­ши­ны де­ре­ва?

41. Хо­зя­ин до­го­во­рил­ся с ра­бо­чи­ми, что они вы­ко­па­ют ему ко­ло­дец на сле­ду­ю­щих усло­ви­ях: за пер­вый метр он за­пла­тит им 4200 руб­лей, а за каж­дый сле­ду­ю­щий метр - на 1300 руб­лей боль­ше, чем за преды­ду­щий. Сколь­ко денег хо­зя­ин дол­жен будет за­пла­тить ра­бо­чим, если они вы­ко­па­ют ко­ло­дец глу­би­ной 11 мет­ров?

42. Хо­зя­ин до­го­во­рил­ся с ра­бо­чи­ми, что они ко­па­ют ко­ло­дец на сле­ду­ю­щих усло­ви­ях: за пер­вый метр он за­пла­тит им 3500 руб­лей, а за каж­дый сле­ду­ю­щий метр - на 1600 руб­лей боль­ше, чем за преды­ду­щий. Сколь­ко денег хо­зя­ин дол­жен будет за­пла­тить ра­бо­чим, если они вы­ко­па­ют ко­ло­дец глу­би­ной 9 мет­ров?

43. В кор­зи­не лежит 45 гри­бов: ры­жи­ки и груз­ди. Из­вест­но, что среди любых 23 гри­бов име­ет­ся хотя бы один рыжик, а среди любых 24 гри­бов хотя бы один груздь. Сколь­ко ры­жи­ков в кор­зи­не?

44. В кор­зи­не лежит 25 гри­бов: ры­жи­ки и груз­ди. Из­вест­но, что среди любых 11 гри­бов име­ет­ся хотя бы один рыжик, а среди любых 16 гри­бов хотя бы один груздь. Сколь­ко ры­жи­ков в кор­зи­не?

45. Спи­сок за­да­ний вик­то­ри­ны со­сто­ял из 25 во­про­сов. За каж­дый пра­виль­ный ответ уче­ник по­лу­чал 7 очков, за не­пра­виль­ный ответ с него спи­сы­ва­ли 10 очков, а при от­сут­ствии от­ве­та да­ва­ли 0 очков. Сколь­ко вер­ных от­ве­тов дал уче­ник, на­брав­ший 42 очка, если из­вест­но, что по край­ней мере один раз он ошиб­ся?

46. На палке от­ме­че­ны по­пе­реч­ные линии крас­но­го, жел­то­го и зе­ле­но­го цвета. Если рас­пи­лить палку по крас­ным ли­ни­ям, то по­лу­чит­ся 5 кус­ков, если по жел­тым ― 7 кус­ков, а если по зе­ле­ным ― 11 кус­ков. Сколь­ко кус­ков по­лу­чит­ся, если рас­пи­лить палку по ли­ни­ям всех трех цве­тов?

47. Улит­ка за день за­пол­за­ет вверх по де­ре­ву на 2 м, а за ночь спол­за­ет на 1 м. Вы­со­та де­ре­ва 11 м. За сколь­ко дней улит­ка до­ползёт от ос­но­ва­ния до вер­ши­ны де­ре­ва?

48. Улит­ка за день за­пол­за­ет вверх по де­ре­ву на 4 м, а за ночь спол­за­ет на 2 м. Вы­со­та де­ре­ва 14 м. За сколь­ко дней улит­ка до­ползёт от ос­но­ва­ния до вер­ши­ны де­ре­ва?

49. Пря­мо­уголь­ник раз­бит на че­ты­ре мень­ших пря­мо­уголь­ни­ка двумя пря­мо­ли­ней­ны­ми раз­ре­за­ми. Пе­ри­мет­ры трёх из них, на­чи­ная с ле­во­го верх­не­го и далее по ча­со­вой стрел­ке, равны 24, 28 и 16. Най­ди­те пе­ри­метр четвёртого пря­мо­уголь­ни­ка.

50. В об­мен­ном пунк­те можно со­вер­шить одну из двух опе­ра­ций:

1) за 2 зо­ло­тых мо­не­ты по­лу­чить 3 се­реб­ря­ных и одну мед­ную;

2) за 5 се­реб­ря­ных монет по­лу­чить 3 зо­ло­тых и одну мед­ную.

У Ни­ко­лая были толь­ко се­реб­ря­ные мо­не­ты. После не­сколь­ких по­се­ще­ний об­мен­но­го пунк­та се­реб­ря­ных монет у него стало мень­ше, зо­ло­тых не по­яви­лось, зато по­яви­лось 50 мед­ных. На сколь­ко умень­ши­лось ко­ли­че­ство се­реб­ря­ных монет у Ни­ко­лая?

51. Пря­мо­уголь­ник раз­бит на че­ты­ре мень­ших пря­мо­уголь­ни­ка двумя пря­мо­ли­ней­ны­ми раз­ре­за­ми. Пе­ри­мет­ры трёх из них, на­чи­ная с ле­во­го верх­не­го и далее по ча­со­вой стрел­ке, равны 24, 28 и 16. Най­ди­те пе­ри­метр четвёртого пря­мо­уголь­ни­ка.

52. В об­мен­ном пунк­те можно со­вер­шить одну из двух опе­ра­ций:

1) за 4 зо­ло­тых мо­не­ты по­лу­чить 5 се­реб­ря­ных и одну мед­ную;

2) за 7 се­реб­ря­ных монет по­лу­чить 5 зо­ло­тых и одну мед­ную.

У Ни­ко­лая были толь­ко се­реб­ря­ные мо­не­ты. После не­сколь­ких по­се­ще­ний об­мен­но­го пунк­та се­реб­ря­ных монет у него стало мень­ше, зо­ло­тых не по­яви­лось, зато по­яви­лось 90 мед­ных. На сколь­ко умень­ши­лось ко­ли­че­ство се­реб­ря­ных монет у Ни­ко­лая?

53. Во всех подъездах дома одинаковое число этажей, а на каждом этаже - одинаковое число квартир. При этом число подъездов дома меньше числа квартир на этаже, число квартир на этаже меньше числа этажей, число подъездов больше одного, а число этажей не более 24. Сколько этажей в доме, если в нем всего 156 квартир?

54. В классе учится 26 учащихся. Несколько из них слушают рок, 14 человек слушают рэп, причем и рок, и рэп слушают всего лишь трое. Известно, что четверо не слушают ни рок, ни рэп. Сколько человек из класса слушают рок?

55. В садке лежат 35 рыб: окуни и плотвички. Известно, что среди любых 21 рыбы имеется хотя бы одна плотвичка, а среди любых 16 рыб - хотя бы один окунь. Сколько плотвичек в садке?

56. На поверхности глобуса маркером проведены 30 параллелей и 24 меридиана. На сколько частей проведенные линии разделили поверхность глобуса? (меридиан - это дуга окружности, соединяющая Северный и Южный полюсы, а параллель - это граница сечения глобуса плоскостью, параллельной плоскости экватора).

57. В доисторическом обменном пункте можно было совершить одну из двух операций:
- за 2 шкуры пещерного льва получить 5 шкур тигра и 1 шкуру кабана;
- за 7 шкур тигра получить 2 шкуры пещерного льва и 1 шкуру кабана.
У Уна, сына Быка, были только шкуры тигра. После нескольких посещений обменного пункта шкур тигра у него не прибавилось, шкур пещерного льва не появилось, зато появилось 80 шкур кабана. На сколько, в итоге, уменьшилось количество шкур тигра у Уна, сына Быка?

58. В войсковой части 32103 имеется 3 вида салата, 2 вида первого блюда, 3 вида второго блюда и на выбор компот или чай. Сколько вариантов обеда, состоящего обязательно из одного салата, одного первого блюда, одного второго блюда и одного напитка, могут выбрать военнослужащие этой войсковой части?

59. Улитка за день заползает вверх по дереву на 5 метров, а за ночь сползает вниз на 3 метра. Высота дерева 17 метров. На какой день улитка впервые доползет до вершины дерева?

60. Сколькими способами можно поставить в ряд три одинаковых желтых кубика, один синий кубик и один зеленый кубик?

61. Произведение шестнадцати идущих подряд натуральных чисел разделили на 11. Чему может быть равен остаток от деления?

62. Каждую минуту бактерия делится на две новые бактерии. Известно, что весь объем трехлитровой банки бактерии заполняют за 4 часа. За сколько секунд бактерии заполняют четверть банки?

63. Список заданий викторины состоял из 36 вопросов. За каждый правильный ответ ученик получал 5 очков, за неправильный ответ с него списывали 11 очков, а при отсутствии ответа давали 0 очков. Сколько верных ответов дал ученик, набравший 75 очков, если известно, что по крайней мере один раз он ошибся?

64. Кузнечик прыгает по прямой дороге длина одного прыжка 1 см. сначала он прыгает 11 прыжков вперед потом 3 назад потом опять 11 прыжков и затем назад 3 прыжка и так далее сколько прыжков он сделает к моменту когда впервые окажется на расстоянии 100 см. от начала.

65. На палке отмечены поперечные линии красного, жёлтого и зелёного цвета. Если распилить палку по красным линиям, то получится 7 кусков, если по жёлтым - 13 кусков, а если по зелёным - 5 кусков. Сколько кусков получится, если распилить палку по линиям всех трёх цветов?

66. В обменном пункте можно совершить одну из двух операций:
за 2 золотых монеты получить 3 серебряных и одну медную;
за 5 серебряных монет получить 3 золотых и одну медную.
У Николая были только серебряные монеты. После нескольких посещений обменного пункта серебряных монет у него стало меньше, золотых не появилось, зато появилось 50 медных. На сколько уменьшилось количество серебряных монет у Николая?

67. Прямоугольник разбит на четыре меньших прямоугольника двумя прямолинейными разрезами.
Периметры трёх из них, начиная с левого верхнего и далее по часовой стрелке, равны 24, 28 и 16. Найдите периметр четвёртого прямоугольника.

68. В обменном пункте можно совершить одну из двух операций:
1) за 4 золотых монеты получить 5 серебряных и одну медную;
2) за 7 серебряных монет получить 5 золотых и одну медную.
У Николы были только серебряные монеты. После посещений обменного пункта серебряных монет у него стало меньше, золотых не появилось, зато появилось 90 медных. На сколько уменьшилось количество серебряных монет?

69. Улитка за день заползает вверх по дереву на 4 м, а за ночь сползает на 2 м. Высота дерева 12 м. За сколько дней улитка доползёт от основания до вершины дерева?

70. Список заданий викторины состоял из 32 вопросов. За каждый правильный ответ ученик получает 5 очков. За неправильный списывали 9, при отсуттвии ответа давали 0 очков.
Сколько верных ответов дал ученик, набравший 75 баллов, если он по крайней мере 2 раза ошибся?

71. Список заданий викторины состоял из 25 вопросов. За каждый правильный ответ ученик получал 7 очков, за неправильный ответ с него списывали 10 очков, а при отсутствии ответа давали 0 очков. Сколько верных ответов дал ученик, набравший 42 очка, если известно, что по крайней мере один раз он ошибся?

72. Хозяин договорился с рабочими, что они выкопают ему колодец на следующих условиях: за первый метр он заплатит им 4200 рублей, а за каждый следующий метр - на 1300 рублей больше, чем за предыдущий. Сколько рублей хозяин должен будет заплатить рабочим, если они выкопают колодец глубиной 11 метров?

73. Прямоугольник разбит на четыре маленьких прямоугольника двумя прямолинейными разрезами. Площади трёх из них начиная с левого верхнего и далее по часовой стрелке равны 18, 12 и 20. Найдите площадь четвёртого прямоугольника.

74. Прямоугольник разбит на четыре маленьких прямоугольника двумя прямолинейными разрезами. Площади трёх из них начиная с левого верхнего и далее по часовой стрелке равны 12, 18 и 30. Найдите площадь четвёртого прямоугольника.

75. В таблице три столбца и несколько строк. В каждую клетку таблицы поставили по натуральному числу так, что сумма всех чисел в первом столбце равна 85, во втором - 77, в третьем - 71, а сумма чисел в каждой строке больше 12, но меньше 15. Сколько всего строк в таблице?

76. Кузнечик прыгает вдоль координатной прямой в любом направлении на единичный отрезок за прыжок. Сколько существует различных точек на координатной прямой, в которых кузнечик может оказаться, сделав 10 прыжков, начиная прыгать из начала координат?

77. Саша пригласил Петю в гости, сказав, что живёт в седьмом подъезде в квартире № 462, а этаж сказать забыл. Подойдя к дому, Петя обнаружил, что дом семиэтажный. На каком этаже живёт Саша? (На всех этажах число квартир одинаково, номера квартир в доме начинаются с единицы.)

78. В обменном пункте можно совершить одну из двух операций:
за 2 золотые монеты получить 3 серебряные и одну медную;
за 7 серебряных монет получить 3 золотые и одну медную.
У Николая были только серебряные монеты. После обменного пункта золотых монет у него не появилось, зато появилось 20 медных. На сколько уменьшилось количество серебряных монет у Николая?

79. Кузнечик прыгает вдоль координатной прямой в любом направлении на единичный отрезок за прыжок. Сколько существует различных точек на координатной прямой, в которых кузнечик может оказаться, сделав 11 прыжков, начиная прыгать из начала координат?

80. На кольцевой дороге расположены четыре бензоколонки: А, Б, В и Г. Расстояние между А и Б - 35 км, между А и В - 20 км, между В и Г - 20 км, между Г и А - 30 км (все расстояния измеряются вдоль кольцевой дороги по кратчайшей дуге). Найдите расстояние (в километрах) между Б и В.

81. В обменном пункте можно совершить одну из двух операций:
за 4 золотые монеты получить 5 серебряных и одну медную;
за 7 серебряных монет получить 5 золотых и одну медную.
У Николая были только серебряные монеты. После обменного пункта серебряных монет у него стало меньше, Золотых не появилось, зато появилось 90 медных. На сколько уменьшилось количество серебряных монет у Николая.

82. Кузнечик прыгает вдоль координатной прямой в любом направлении на единичный отрезок за прыжок. Сколько существует точек на координатной прямой, в которых кузнечик может оказаться, сделав ровно 8 прыжков, начиная прыгать из начала координат?

83. В обменном пункте можно совершить одну из двух операций:
за 5 золотых монет получить 4 серебряные и одну медную;
за 10 серебряных монет получить 7 золотых и одну медную.
У Николая были только серебряные монеты. После обменного пункта серебряных монет у него стало меньше, золотых не появилось, зато появилось 60 медных. На сколько уменьшилось количество серебряных монет у Николая?

84. В обменном пункте можно совершить одну из двух операций:
за 5 золотых монет получить 6 серебряных и одну медную;
за 8 серебряных монет получить 6 золотых и одну медную.
У Николая были только серебряные монеты. После обменного пункта серебряных монет у него стало меньше, золотых не появилось, зато появилось 55 медных. На сколько уменьшилось количество серебряных монет у Николая?

85. Во всех подъездах дома одинаковое число этажей, и па всех этажах одинаковое число квартир. При этом число этажей в доме больше числа квартир на этаже, число квартир на этаже больше числа подъездов, а число подъездов больше одного. Сколько этажей в доме, если всего в нём 105 квартир?

86. В обменном пункте можно совершить одну из двух операций:
1) за 3 золотых монеты получить 4 серебряных и одну медную;
2) за 7 серебряных монет получить 4 золотых и одну медную.
У Николы были только серебряные монеты. После посещений обменного пункта серебряных монет у него стало меньше, золотых не появилось, зато появилось 42 медных. На сколько уменьшилось количество серебряных монет у Николы?

ОТВЕТЫ

Яковлева Наталья Сергеевна
Должность: учитель математики
Учебное заведение: МКОУ "Бунинская СОШ"
Населённый пункт: село Бунино, Солнцевский район, Курская область
Наименование материала: статья
Тема: "Методы решения заданий №20 ЕГЭ по математике базовый уровень"
Дата публикации: 05.03.2018
Раздел: полное образование

Единый государственный экзамен является на данный момент единственной

формой итоговой аттестации выпускников средней школы. А получение

аттестата о среднем образовании не возможно без успешной сдачи ЕГЭ по

математике. Математика является не только важным учебным предметом, но

и достаточно сложным. Математическими способностями обладают далеко

не все дети, а от успешной сдачи экзамена зависит их дальнейшая судьба.

Учителя выпускных классов снова и снова задают вопрос: «Как помочь

школьнику при подготовке к ЕГЭ и успешно его сдать?». Для того, чтобы

выпускник получил аттестат достаточно сдать математику базового уровня. А

успешность сдачи экзамена напрямую связана с тем, как учитель владеет

методикой решения различных задач. Вашему вниманию предлагаю примеры

решения задания №20 математика базовый уровень ФИПИ 2018 под

редакцией М.В. Ященко.

1 .На ленте по разные стороны от середине отмечены две полосы: синяя и

красная. Если ленту разрезать по красной полосе, то одна часть будет на 5 см

длиннее другой. Если ленту разрезать по синей полосе, то одна часть будет на

15 см длиннее другой. Найдите расстояние между красной и синей

полосами.

Решение:

Пусть а см расстояние от левого конца ленты до синей полосы, в см

расстояние от правого конца ленты до красной полосы, с см расстояние

между полосами. Известно, что если ленту разрезать по красной полосе, то

одна часть на 5 см длиннее другой, то есть а + с – в =5. Если разрезать по

синей полосе, то одна часть будет длиннее другой на 15 см, значит, в +с –

а=15. Сложим два равенство почленно: а+с-в+в+с-а=20, 2с=20, с=10.

2 . Среднее арифметическое 6 различных натуральных чисел равно 8. На

сколько нужно увеличить наибольшее из этих чисел, чтобы среднее

арифметическое стало на 1 больше.

Решение: Так как среднее арифметическое 6 натуральных чисел равно 8,

значит, сумма этих чисел равна 8*6=48. Среднее арифметическое чисел

увеличилось на 1 и стало равно 9, а количество чисел не изменилось, значит,

сумма чисел стане равной 9*6=54. Чтобы найти на сколько увеличилось одно

из чисел, нужно найти разность 54-48=6.

3. Клетки таблицы 6х5 раскрашены в черные и белые цвета. Пар соседних

клеток разного цвета 26, пар соседних клеток черного цвета 6. Сколько пар

соседних клеток белого цвета.

Решение:

В каждой горизонтали образуется 5 пар соседних клеток, значит, по

горизонтали всего будет 5*5=25 пар соседних клеток. По вертикали

образуется 4 пары соседних клеток, то есть всего пар соседних клеток по

вертикали будет 4*6=24. Всего образуется 24+25=49 пар соседних клеток. Из

них разного цвета 26 пар, черного 6 пар, следовательно белых пар будет 49-

26-6 = 17 пар.

Ответ: 17 .

4. На прилавке цветочного магазина стоят три вазы с розами: белая, синяя и

красная. Слева от красной вазы находится 15 роз, справа от синей вазы 12

роз. Всего в вазах 22 розы. Сколько роз в белой вазе?

Решение: Пусть х роз находится в белой вазе, у роз – в синей, z роз – в

красной. По условию задачи в вазах 22 розы, то есть х+у+ z=22. Известно,

что слева от красной вазы, то есть в синей и белой 15 роз, значит, х+у=15. А

справа от синей вазы, то есть в белой и красной вазах 12 роз, значит х+ z= 12.

Получили:

Прибавим почленно 2-ое и 3-ье равенства: х+у+х+ z=27 или 22 +х=27, х=5.

5 .Маша и Медведь съели 160 печений и банку варенья, начав и закончив

одновременно. Сначала Маша ела варенье, а Медведь печенья, но в какой-то

момент они поменялись. Медведь и то и другое ест в 3 раза быстрее Маши.

Сколько печений съел Медведь, если варенья они съели поровну.

Решение: Так как Маша и Медведь начали есть печенья и варенье

одновременно и закончили одновременно, причем ели один продукт, а затем

другой, и по условию задачи Медведь ест и то и другое в 3 раза быстрее, чем

Маша, значит Медведь поглощал еду в 9 раз быстрее Маши. Тогда пусть х

печений съела Маша, а Медведь 9х печений. Известно, что всего они съели

160 печений. Получим: х+9х=160, 10х=160, х=16, значит, медведь съел

16*9=144 печенья.

6. Из книги выпало несколько идущих подряд листов. Номер последней

страницы перед выпавшими листами 352. Номер первой страницы после

выпавших листов записывается теми же цифрами, но в другом порядке.

Сколько листов выпало?

Решение: Пусть х листов выпало, тогда количество выпавших страниц 2х, то

есть четное число. Номер первой выпавшей страницы 353. Разность между

номером первой выпавшей страницы и первой страницы после выпавших

должно быть четным числом, значит, номер после выпавших листов будет

523. Тогда количество выпавших листов будет равно (523-353):2=85.

7. Про натуральные числа А,В,С известно, что каждое из них больше 5, но

меньше 9. Загадали натуральное число, затем умножили на А, прибавили В и

вычли С. Получили 164. Какое число было задумано?

Решение: Пусть х загаданное натуральное число, тогда Ах+В-С=164, Ах=

164 – (В-С), так как числа А,В,С больше 5, но меньше 9, то -2≤В-С≤2,

значит, Ах= 166; 165; 164;163;162. Из чисел 6,7,8 только 6 является

Задание №20 ЕГЭ по математике содержит задачу на сообразительность. Задачи в этом разделе более интуитивно понятно, нежели в 19 задании ЕГЭ, но тем не менее достаточно сложны для обычного школьника. Итак, перейдем к рассмотрению типовых вариантов.

Разбор типовых вариантов заданий №20 ЕГЭ по математике базового уровня

Первый вариант задания (демонстрационный вариант 2018)

  • за 2 золотых монеты получить 3 серебряных и одну медную;
  • за 5 серебряных монет получить 3 золотых и одну медную.

У Николая были только серебряные монеты. После нескольких посещений обменного пункта серебряных монет у него стало меньше, золотых не появилось, зато появилось 50 медных. На сколько уменьшилось количество серебряных монет у Николая?

Алгоритм выполнения:
  1. Ввести условные обозначения.
  2. Записать данные задачи с помощью условных обозначений.
  3. Логически рассуждая определить неизвестное.
Решение:

По условию золотых монет не появилось, значит все полученные после осуществления второй операции золотые монеты, Николай обменял с помощью первой операции. Золотые монеты можно менять только по 2 штуки, следовательно, вторых операций было четное число.

Введем обозначение, пусть вторых операций было 2n(число всегда четное).

Если применить вторую операцию получим:

Все золотые монеты были обменяны в ходе первой операции. За одну операцию можно обменять сразу 2 золотые монеты, значит, всего операций будет совершено (3 · 2n)/2 = 3 n. То есть

3 · 2n золотых обменяли на 3· 3n серебряных + 3n медных.

Или после преобразования:

Сопоставим результаты первой и второй операции:

5 · 2n серебряных обменяли на 3 · 2n золотых + 2n медных.

3 · 2n золотых обменяли на 9n серебряных + 3n медных

5 · 2n серебряных обменяли на 9n серебряных + 3n медных+2n медных

10 n серебряных обменяли на 9n серебряных + 5n медных

Если, обменяв 10 n серебряных монет, получим 9 n серебряных монет, то количество серебряных монет у Николая уменьшилось на n. Из последнего выражения видно, что Николай получил 5n медных монет, а по условию появилось 50 медных, то есть 5n = 50.

Второй вариант задания

Маша и Медведь съели 100 печений и банку варенья, начав и закончив одновременно. Сначала Маша ела варенье, а Медведь - печенья, но в какой-то момент они поменялись. Медведь и то, и другое ест в три раза быстрее Маши. Сколько печений съел Медведь, если варенья они съели поровну?

Алгоритм выполнения:
  1. Сопоставить результаты.
  2. Найти неизвестное.
Решение:
  1. Так как варенье и Маша, и Медведь съели поровну, и при этом Медведь ел варенье в 3 раза быстрее, то Маша ела варенье (свою половину) в 3 раза дольше, чем Медведь (такую же половину).
  2. Тогда получается, что Медведь ел печенья в 3 раза дольше Маши и к тому же ел их в 3 раза быстрее, то есть, на одно съеденное Машей печенье приходилось 3∙3=9 печений, съеденных Медведем.
  3. В сумме эти печенья составляют 1+9=10 и таких сумм в 100 печеньях ровно 100:10 = 10.
  4. Значит, Маша съела 10 печений, а Медведь 9∙10=90.

Третий вариант задания

Маша и Медведь съели 51 печенье и банку варенья, начав и закончив одновременно. Сначала Маша ела варенье, а Медведь - печенья, но в какой-то момент они поменялись. Медведь и то, и другое ест в четыре раза быстрее Маши. Сколько печений съел Медведь, если варенья они съели поровну?

Алгоритм выполнения:
  1. Определить, кто и во сколько раз дольше ел печенье.
  2. Определить, кто и во сколько раз дольше ел варенье.
  3. Сопоставить результаты.
  4. Найти неизвестное.
Решение:
  1. Так как варенье и Маша, и Медведь, съели поровну, и при этом Медведь ел варенье в 4 раза быстрее, то Маша ела варенье (свою половину) в 4 раза дольше, чем Медведь (такую же половину).
  2. Тогда получается, что Медведь ел печенья в 4 раза дольше Маши и к тому же ел их в 4 раза быстрее, то есть, на одно съеденное Машей печенье приходилось 4∙4=16 печений, съеденных Медведем.
  3. В сумме эти печенья составляют 1+16=17 и таких сумм в 51 печеньях ровно 51:17 = 3.
  4. Значит, Маша съела 3 печенья, а Медведь 3∙16=48.

Четвертый вариант задания

Если бы каждый из двух сомножителей увеличили на 1, их произведение увеличилось бы на 11. На самом деле каждый из двух сомножителей увеличили на 2. На сколько увеличилось произведение?

Алгоритм выполнения:
  1. Ввести условные обозначения.
  2. Преобразовать полученное выражение.
  3. Найти неизвестное.
Решение:

При увеличении этих сомножителей на 1 их произведение возрастает на 11, то есть,

Теперь аналогично вычислим, на сколько увеличится произведение, если сомножители увеличить на 2 и подставим уже известное нам a + b = 10:

Пятый вариант задания

Если бы каждый из двух сомножителей увеличили на 1, их произведение увеличилось бы на 3. На самом деле каждый из двух сомножителей увеличили на 5. На сколько увеличилось произведение?

Алгоритм выполнения:
  1. Ввести условные обозначения.
  2. Записать первое условие с помощью условных обозначений.
  3. Преобразовать полученное выражение.
  4. Записать с помощью условных обозначений второе условие.
  5. Преобразовать полученное выражение.
  6. Найти неизвестное.
Решение:

Пусть первый сомножитель равен a, а второй b, их произведение равно ab.

При увеличении этих сомножителей на 1 их произведение возрастает на 3, то есть,

Перенесем произведение ab в левую часть с противоположным знаком и раскроем скобки перемножив.

Теперь аналогично вычислим, на сколько увеличится произведение, если сомножители увеличить на 5 и подставим уже известное нам a + b = 2:

Вариант двадцатого задания 2017

Прямоугольник разбит на четыре меньших прямоугольника двумя прямолинейными отрезками. Периметры трёх из них, начиная с левого верхнего и далее по часовой стрелке, равны 24, 28 и 16. Найдите периметр четвёртого прямоугольника.

Перерисуем прямоугольник в удобном для нас виде:

Теперь составим уравнения с помощью формулы периметра прямоугольника:

Вариант двадцатого задания 2019 года (1)

Список заданий викторины состоял из 25 вопросов. За каждый правильный ответ ученик получал 7 очков, за неправильный ответ с него списывали 10 очков, а при отсутствии ответа давали 0 очков. Сколько верных ответов дал ученик, набравший 42 очка, если известно, что по крайней мере один раз он ошибся?

Алгоритм выполнения
  1. Составляем комбинации правильных и неправильных ответов и определяем кол-во баллов в них, например: 1) 1 прав+1 неправ=7–10=–3 балла; 2) 2 прав+1неправ=2·7–10=4 балла и т.д.
  2. Из баллов за прав.ответы и баллов за их комбинации «набираем» 42 балла. Подсчитываем кол-во вопросов, которые при этом были заданы.
  3. Оставшуюся разницу между полученным числом вопросов и данными 25-ю вопросами определяем как те, на которые не было дано ответа.
  4. Делаем проверку полученного результата.
Решение:

Введем обозначения: прав.ответ – 1П, неправ.ответ – 1Н.

Задаем комбинации и определяем кол-во баллов, которое при этом будет начислено:

1П=7 баллов

1П+1Н=7–10=–3 б.

2П+1Н=2·7–10=4 б.

3П+1Н=3·7–10=11 б.

Суммируем баллы, которые можно при этом получить: 7+ (–3)+4+11=19. Это явно мало. И гарантированно можно добавить еще 11: 19+11=30. Чтобы «добрать» до 42 баллов, нужно далее добавить 12 баллов, которые набираются тройным вхождением 4-х баллов. В целом получаем:

7+(–3)+4+11+11+3·4=42.

Распишем полученную комбинацию слагаемых в виде ответов:

1П+(1П+1Н)+(2П+1Н)+(3П+1Н)+(3П+1Н)+3·(2П+1Н)=1П+1П+1Н+2П+1Н+3П+1Н+3П+1Н+6П+3Н=16П+7Н (ответов).

16+7=23 ответа. 25–23=2 ответа, за которые было получено по 0 баллов, т.е. это вопросы, оставшиеся без ответов.

Итак, по нашим подсчетам верных ответов было дано 16.

Проверим это:

16 ответов по 7 б. + 7 ответов по (–10) б. + 2 ответа по 0 б. = 16·7–7·10+2·0=112–70+0=42 (балла).

Вариант двадцатого задания 2019 года (2)

В таблице три столбца и несколько строк. В каждую клетку таблицы вписали по натуральному числу так, что сумма всех чисел в первом столбце равна 103, во втором – 97, в третьем – 93, а сумма чисел в каждой строке больше 21, но меньше 24. Сколько всего строк в таблице?

Алгоритм выполнения
  1. Находим общую сумму для всех чисел в таблице (сложив суммы для каждого из 3-х столбцов).
  2. Определяем диапазон допустимых значений для сумм чисел в каждой строке.
  3. Разделив общую сумму сначала на наименьшую сумму чисел в каждой строке, а затем на наибольшую, получаем искомое кол-во строк.
Решение:

Общая сумма чисел в таблице равна: 103+97+93=293.

Поскольку по условию суммы чисел в каждой строке составляют >21, но <24, то кол-во строк X может быть равным меньше, чем 293:21≈13,95, и больше, чем 293:24≈12,21. Т.е.: 12,21 < X < 13,95. Единственное целое число в полученном диапазоне – 13. Значит, искомое кол-во строк равно 13.

Вариант двадцатого задания 2019 года (3)

В доме всего восемнадцать квартир с номерами от 1 до 18. В каждой квартире живет не менее одного и не более трех человек. В квартирах с 1-й по 13-ю включительно живет суммарно 15 человек, а в квартирах с 11-й по 18-ю включительно живет суммарно 20 человек. Сколько всего человек живет в этом доме?

Алгоритм выполнения
  1. Определяем максимальное кол-во живущих в 11–13-й квартирах, используя данные о том, сколько человек живет в 1–13-й квартирах.
  2. Находим минимальное число жильцов 11–13-й квартир, учитывая данные о живущих в 11–18-й квартирах.
  3. Сопоставляет данные, полученные в пп.1–2, получаем точное кол-во жильцов этих квартир №№11–13.
  4. Находим кол-во живущих в квартирах 1–10-й и 14–18-й.
  5. Вычисляем общее число жильцов дома.
Решение:

В первых 13 квартирах (с 1-й по 13-ю) живет 15 человек. Это означает, что в 11-ти квартирах живет по 1 человеку плюс в 2-х квартирах по 2 человека (11·1+2·2=15). Следовательно, в 11–13-й (т.е. в 3-х) квартирах проживает не менее 3-х и не более 5 (1+2+2) человек.

Во вторых 8 квартирах (11-й по 18-ю) проживает 20 человек. При этом с 14-й по 18-ю квартиры (т.е. в 5 квартирах) не может проживать более чем 5·3=15 человек. А следовательно, в 11-13-й квартирах живет не менее, чем 20–15=5 человек.

Т.е. с одной стороны в 11-13-й квартирах должно жить не более 5 человек, а с другой – не менее 5. Вывод: в этих квартирах живет ровно 5 человек, т.к. других допустимых для обоих случаев значений тут нет.

Тогда получаем: в 1–10-й квартирах живет 15–5=10 человек, в 14–18-й – 20–5=15 человек. Всего в доме проживает: 10+5+15=30 человек.

Вариант двадцатого задания 2019 года (4)

В обменном пункте можно совершить одну из двух операций:

  • за 4 золотых монеты получить 5 серебряных и одну медную;
  • за 7 серебряных монет получить 5 золотых и одну медную.

У Николая были только серебряные монеты. После нескольких посещений обменного пункта серебряных монет у него стало меньше, золотых не появилось, зато появилось 45 медных. На сколько уменьшилось количество серебряных монет у Николая?

Алгоритм выполнения
  1. Определяем кол-во серебряных монет, которые необходимы Николаю для совершения двойного обмена так, чтобы у него не появились золотые монеты. Двойной обмен – это обмен сначала серебряных монет на золотые и медные, а затем золотые на серебряные и медные.
  2. Определяем кол-во разных монет, которые появятся у Николая в результате 1 двойного обмена.
  3. Вычисляем кол-во двойных обменов, которые необходимо совершить, чтобы появилось 45 медных монет.
  4. Находим кол-во серебряных монет, которые должен был иметь Николай изначально, чтобы совершить нужное кол-во обменов, и которые получил в результате всех обменов.
  5. Определяем искомую разницу.
Решение:

Совершить 1-й обмен Николай должен по 2-й схеме, т.к. у него есть только серебряные монеты. Для того же, чтобы в результате у него не оказалось золотых монет, нужно найти минимальное кратное для 5 золотых, которые он получит, и 4 золотых, которые у него за 1 раз могут принять в полном объеме (без остатка). Это – число 20.

Соответственно, чтобы получить 20 золотых монет, у Николая должно быть 20:5=4 комплекта серебряных монет по 7 штук. Значит, первоначально их у него должно быть 4·7=28. И при этом Николай получает еще и 1·4=4 медных монеты.

Совершая обмен, Николай отдает 20:4=5 комплектов золотых медалей. Взамен он получает 5·5=25 серебряных монет и 1·5=5 медных монет.

Т.о., в результате одного обмена у Николая появится 25 серебряных монет и 4+5=9 медных монет. Поскольку в итоге у Николая оказалось 45 медных монет, значит, было совершено 45:9=5 двойных обменов.

Если в результате 1 двойного обмена у Николая оказалось 25 серебряных монет, то после 5 таких обменов у него их окажется 25·5=125 штук. А первоначально он должен был для этого иметь 28·5=140 серебряных монет. Следовательно, их количество у Николая уменьшилось на 140–125=15 штук.

Вариант двадцатого задания 2019 года (5)

Во всех подъездах дома одинаковое число этажей, и на всех этажах одинаковое число квартир. При этом число этажей в доме больше числа квартир на этаже, число квартир на этаже больше числа подъездов, а число подъездов больше одного. Сколько этажей в доме, если всего в нем 357 квартир?

Алгоритм выполнения
  1. Определяем уравнение для определения кол-ва квартир в доме всего через параметры, заявленные в условии (т.е. через кол-во квартир на этаже и т.д.).
  2. Раскладываем 357 на множители.
  3. Находим соответствие полученных множителей конкретным параметрам, сходя из условия о том, какой из параметров больше или меньше прочих.
Решение:

Т.к. на всех этажах одинаковое кол-во квартир (Х), по всех подъездах одинаковое кол-во этажей (Y), то обозначив кол-во подъездов через Z, можем записать: 357=X·Y·Z.

Разложим 357 на простые множители. Получим: 357=3·7·17·1. Причем это единственный вариант расклада. Т.к. Y>X>Z>1, то единицу в раскладе не учитываем и определяем, что Z=3, X=7, Y=17.

Поскольку кол-во этажей было обозначено через Y, то искомое число – 17.

Вариант двадцатого задания 2019 года (6)

Из десяти стран семь подписали договор о дружбе ровно с тремя странами, а каждая из оставшихся трех – ровно с семью. Сколько всего было подписано договоров?

Алгоритм выполнения
  1. Подсчитываем кол-во договоров, подписанных 7-ю странами.
  2. Определяем кол-во договоров, которые подписали 3 оставшиеся страны.
  3. Находим общее кол-во подписанных договоров. Делим его на 2, т.к. договоры двусторонние.
Решение:

Первые 7 стран подписали договоры с 3 странами, т.е. на этих договорах поставлено 7·3=21 подпись. Аналогично остальные 3 страны при оформлении договоров с 7-ю странами поставили 3·7=21 подпись. Значит, всего поставлено 21+21=42 подписи.

Т.к. все договоры двусторонние, то это значит, что на каждом из них зафиксировано 2 подписи. Следовательно, договоров вдвое меньше, чем подписей, т.е. 42:2=21 договор.

Вариант двадцатого задания 2019 года (7)

На поверхности глобуса фломастером проведены 13 параллелей и 25 меридианов. На сколько частей проведенные линии разделили поверхность глобуса?

Меридиан – это дуга окружности, соединяющая Северный и Южный полюсы. Параллель – это окружность, лежащая в плоскости, параллельной плоскости экватора.

Алгоритм выполнения
  1. Доказываем, что параллели делят глобус на 13+1 часть.
  2. Доказываем, что меридианы делят глобус на 25 частей.
  3. Определяем кол-во частей, на которые в целом разделен глобус, как произведение найденных чисел.
Решение:

Если всякая параллель – это окружность, то она является замкнутой линией. А это означает, что 1-я параллель делит глобус на 2 части. Далее 2-я параллель обеспечивает деление на 3 части, 3-я – на 4 и т.д. В итоге 13 параллелей разделят глобус на 13+1=14 частей.

Меридиан является дугой окружности, соединяющей полюса, т.е. замкнутой линией она не является и глобус на части не делит. А вот 2 меридиана уже делят, т.е. 2 меридиана обеспечивают деление на 2 части, далее 3-й меридиан добавляет 3-ю часть, 4-й – 5-ю часть и т.д. Значит, в конечном счете, 25 меридианов создает на глобусе 25 частей.

Всего частей на глобусе получается: 14·25=350 частей.

Вариант двадцатого задания 2019 года (8)

В корзине лежит 30 грибов: рыжики и грузди. Известно, что среди любых 12 грибов имеется хотя бы один рыжик, а среди любых 20 грибов – хотя бы один груздь. Сколько рыжиков в корзине?

Алгоритм выполнения
  1. Определяем кол-во груздей среди 12 грибов и рыжиков среди 20 грибов.
  2. Доказываем, что имеется единственно верное число, отображающее кол-во рыжиков. Фиксируем его в ответе.
Решение:

Если среди 12 грибов есть как минимум 1 рыжик, значит, груздей здесь не более 11. Если среди 20 грибов имеется не менее 1 груздя, то тут не более 19 рыжиков.

Это означает, что если груздей не может быть больше 11, то рыжиков не может быть меньше 30–11=19 штук. Т.е. рыжиков с одной стороны не больше 19, а с другой – не меньше 19. Следовательно, рыжиков может быть только ровно 19.

Вариант двадцатого задания 2019 года (9)

Если бы каждый из двух множителей увеличили на 1, то их произведение увеличилось бы на 3. На сколько увеличится произведение этих множителей, если каждый из них увеличить на 5?

Алгоритм выполнения
  1. Вводим обозначения для множителей. Это позволит выразить и первоначальное произведение (до увеличения множителей).
  2. Составляем уравнение для ситуации, когда множители увеличены на 1. Выполняем преобразования. Получаем новое выражение, отображающее связь между первоначальными множителями.
  3. Составляем уравнение для ситуации, когда множители увеличены на 5. Выполняем преобразования. Вводим в уравнение выражение, полученное в п.2, находим искомую разницу.
Решение:

Пусть 1-й множитель равен х, 2-й – у. Тогда их произведение – ху.

После того, как множители увеличены на 1, получаем:

(х+1)(у+1)=ху+3

ху +у+х+1= ху +3

После увеличения множителей на 5 имеем:

(х+5)(у+5)=ху+N, где N – искомая разница произведений.

Выполняем преобразования:

ху+5у+5х+25=ху+N

N= ху +5у+5х+25– ху

Т.к. выше уже определено, что х+у=2, то получим:

Вариант двадцатого задания 2019 года (10)

Саша пригласил Петю в гости, сказав, что живет в седьмом подъезде в квартире № 462, а этаж сказать забыл. Подойдя к дому, Петя обнаружил, что дом семиэтажный. На каком этаже живет Саша? (На всех этажах число квартир одинакова, нумерация квартир в доме начинается с единицы.)

Алгоритм выполнения
  1. Способом подбора определяем кол-во квартир на площадке. Это должно быть такое число, чтобы номер квартиры оказался большим, чем кол-во квартир в 6-ти подъездах, однако меньшим, чем кол-во квартир в 7-ми.
  2. Определяем кол-во квартир в 6-ти подъездах. От 462 отнимаем это кол-во и делим на число квартир на площадке. Так узнаем искомый номер этажа. Примечание: 1) если получено целое число, то искомый номер этажа на 1 больше, чем вычисленное значение; 2) если получено дробное число, то номером этажа будет округленный в большую сторону результат.
Решение:

Ищем кол-во квартир на площадке, проверяя число за числом.

Предположим, что это кол-во равно 3. Тогда получим, что в 7 подъездах на 6 этажах имеется 7·6·3=126 квартир,

а в 7 подъездах на 7 этажах 7·7·3=147 квартир.

Квартира №462 точно не попадает в диапазон квартир №№126–147.

Аналогично проверяя числа 4, 5 и т.д., придем к числу 10. Докажем, что именно оно подходит:

в 7 подъездах на 6 этажах находится 7·6·10=420 квартир,

в 7 подъездах на 7 этажах: 7·7·10=490 квартир. Поскольку 420<462<490, то условие задания выполнено.

Для того чтобы попасть в квартиру №462, нужно пройти мимо 462–420=42 квартир. Т.к. на каждой площадке находится 10 квартир, то 42:10=4,2 этажей для этого нужно преодолеть. 4,2 означает, что 4 этажа нужно пройти полностью и подняться на 5-й. Т.о., искомый этаж – 5-й.

Задача №5922.

Хозяин договорился с рабочими, что они копают колодец на следующих условиях: за первый метр он заплатит им 3500 рублей, а за каждый следующий метр – на 1600 рублей больше, чем за предыдущий. Сколько денег хозяин должен будет заплатить рабочим, если они выкопают колодец глубиной 9 метров?

Так как оплата каждого следующего метра отличается от оплаты предыдущего на одно и то же число, перед нами .

В этой прогрессии - плата за первый метр, - разница в оплате каждого последующего метра, - количество рабочих дней.

Сумма членов арифметической прогрессии находится по формуле:

Подставим данные задачи в эту формулу.

Ответ: 89100.

Задача №5943.

В об­мен­ном пунк­те можно со­вер­шить одну из двух опе­ра­ций:

· за 2 зо­ло­тые мо­не­ты по­лу­чить 3 се­реб­ря­ные и одну мед­ную;

· за 5 се­реб­ря­ных монет по­лу­чить 3 зо­ло­тые и одну мед­ную.

У Ни­ко­лая были толь­ко се­реб­ря­ные мо­не­ты. После не­сколь­ких по­се­ще­ний об­мен­но­го пунк­та се­реб­ря­ных монет у него стало мень­ше, зо­ло­тых не по­яви­лось, зато по­яви­лось 100 мед­ных. На сколь­ко умень­ши­лось ко­ли­че­ство се­реб­ря­ных монет у Ни­ко­лая ?

Задача №5960.

Кузнечик прыгает вдоль координатной прямой в любом направлении на единичный отрезок за прыжок. Сколько существует различных точек на координатной прямой, в которых кузнечик может оказаться, сделав ровно 5 прыжков, начиная прыгать из начала координат?

Если кузнечик сделает пять прыжков в одном направлении (вправо или влево), то он окажется в точках с координатами 5 или -5:

Заметим, что кузнечик может прыгать и вправо и влево. Если он сделает 1 прыжок вправо и 4 прыжка влево (в сумме 5 прыжков), то окажется в точке с координатой -3. Аналогично, если кузнечик сделает 1 прыжок влево и 4 прыжка вправо (в сумме 5 прыжков), то окажется в точке с координатой 3:

Если кузнечик сделает 2 прыжка вправо и 3 прыжка влево (в сумме 5 прыжков), то окажется в точке с координатой -1. Аналогично, если кузнечик сделает 2 прыжка влево и 3 прыжка вправо (в сумме 5 прыжков), то окажется в точке с координатой 1:


Заметим, что если общее количество прыжков нечетное, то в начало координат кузнечик не вернется, то есть он сможет попасть только в точки с нечетными координатами:


Этих точек всего 6.

Если бы количество прыжков было четным, то кузнечик смог бы вернуться в начало координат и все точки на координатной прямой, в которые он мог бы попасть имели бы четные координаты.

Ответ: 6

Задача №5990

Улитка за день залезает вверх по дереву на 2 м, а за ночь сползает на 1 м. Высота дерева 9 м. За сколько дней улитка доползет до вершины дерева?

Заметим, что в этой задаче следует различать понятие "сутки" и понятие "день".

В задаче спрашивается именно за сколько дней улитка доползет до вершины дерева.

За один день улитка поднимается на 2 м, а за одни сутки улитка поднимается на 1 м (за день поднимается на 2 м, а потом за ночь спускается на 1 м).

За 7 суток улитка поднимается на 7 метров. То есть утром 8-го дня ей останется доползти до вершины 2 м. И за восьмой день она преодолеет это расстояние.

Ответ: 8 дней.

Задача №6010.

Во всех подъездах дома одинаковое число этажей, а на каждом этаже одинаковое число квартир. При этом число этажей в доме больше числа квартир на этаже, число квартир на этаже больше числа подъездов, а число подъездов больше одного. Сколько этажей в доме, если всего в нём 105 квартир?

Чтобы найти число квартир в доме, нужно число квартир на этаже ( ) умножить на число этажей ( ) и умножить на число подъездов ( ).

То есть нам нужно найти ( ), исходя из следующих условий:

(1)

Последнее неравенство отражает условие "число этажей в доме больше числа квартир на этаже, число квартир на этаже больше числа подъездов, а число подъездов больше одного".

То есть ( ) - самое больше число.

Разложим 105 на простые множители:

С учетом условия (1), .

Ответ: 7.

Задача №6036.

В корзине лежат 30 грибов: рыжики и грузди. Известно, что среди любых 12 грибов имеется хотя бы один рыжик, а среди любых 20 грибов хотя бы один груздь. Сколько рыжиков в корзине?

Так как среди любых 12 грибов имеется хотя бы один рыжик (или больше) число груздей должно быть меньше или равно чем .

Отсюда следует, что число рыжиков больше или равно чем .

Так как среди любых 20 грибов хотя бы один груздь (или больше), число рыжиков должно быть меньше или равно чем

Тогда получили, что с одной стороны, число рыжиков больше или равно чем 19 , а с другой - меньше или равно чем 19 .

Следовательно, число рыжиков равно 19.

Ответ: 19.

Задача №6047.

Саша пригласил Петю в гости, сказав, что живёт в седьмом подъезде в квартире № 333, а этаж сказать забыл. Подойдя к дому, Петя обнаружил, что дом девятиэтажный. На каком этаже живёт Саша? (На каждом этаже число квартир одинаково, номера квартир в доме начинаются с единицы.)

Пусть на каждом этаже квартир.

Тогда число квартир в первых шести подъездах равно

Найдем максимальное натуральное значение , удовлетворяющее неравенству ( - номер последней квартиры в шестом подъезде, и он меньше, чем 333.)

Отсюда

Номер последней квартиры в шестом подъезде -

Седьмой подъезд начинается с 325-й квартиры.

Следовательно, 333 квартира находится на втором этаже.

Ответ: 2

Задача №6060.

На поверхности глобуса фломастером проведены 17 параллелей и 24 меридиана. На сколько частей проведённые линии разделяют поверхность глобуса? Меридиан – это дуга окружности, соединяющая Северный и Южный полюса. параллель – это окружность, лежащая в плоскости, параллельной плоскости экватора .

Представим себе арбуз, который мы разрезаем на кусочки.

Сделав два разреза от верхней точки к нижней (проведя два меридиана), мы разрежем арбуз на две дольки. Следовательно, проведя 24 разреза (24 меридиана) мы разрежем арбуз на 24 дольки.

Теперь будем разрезать каждую дольку.

Если мы сделаем 1 поперечный разрез (параллель), то разрежем одну дольку на 2 части.

Если мы сделаем 2 поперечных разреза (параллели), то разрежем одну дольку на 3 части.

Значит, сделав 17 разрезов мы разрежем одну дольку на 18 частей.

Итак, мы разрезали 24 дольки на 18 частей, и получили куска.

Следовательно, 17 параллелей и 24 меридиана разделяют поверхность глобуса на 432 части.

Ответ: 432.

Задача №6069

На палке отмечены поперечные линии красного, жёлтого и зелёного цвета. Если распилить палку по красным линиям, получится 5 кусков, если по жёлтым – 7 кусков, а если по зелёным – 11 кусков. Сколько кусков получится, если распилить палку по линиям всех трёх цветов?

Если сделать 1 разрез, то получится 2 куска.

Если сделать 2 разреза, то получится 3 куска.

В общем случае: если сделать разрезов, то получится кусок.

Обратно: чтобы получить кусков, нужно сделать разрез.

Найдем общее количество линий, по которым разрезали палку.

Если распилить палку по красным линиям, получится 5 кусков - следовательно, красных линий было 4;

если по жёлтым – 7 кусков - следовательно, желтых линий было 6;

а если по зелёным – 11 кусков - следовательно, зеленых линий было 10.

Отсюда общее количество линий равно . Если распилить палку по всем линиям, то получится 21 кусок.

Ответ: 21.

Задача №9626.

На кольцевой дороге расположены четыре бензоколонки: A, Б, B, и Г. Расстояние между A и Б – 50 км, между A и В – 40 км, между В и Г – 25 км, между Г и A – 35 км (все расстояния измеряются вдоль кольцевой дороги в кратчайшую сторону). Найдите расстояние между Б и В.

Посмотрим, как могут быть расположены бензоколонки. Попробуем расположить их так:


При таком расположении расстояние между Г и А не может быть равно 35 км.

Попробуем так:


При таком расположении расстояние между А и В не может быть 40 км.

Рассмотрим такой вариант:


Этот вариант удовлетворяет условию задачи.

Ответ: 10.

Задача №10041.

Список заданий викторины состоял из 25 вопросов. За каждый правильный ответ ученик получал 7 очков, за неправильный ответ с него списывали 9 очков, а при отсутствии ответа давали 0 очков. Сколько верных ответов дал ученик, набравший 56 очков, если известно, что по крайней мере один раз он ошибся?

Пусть ученик дал правильных ответов и неправильных ( ). Так как возможно были еще вопросы, на которые он на ответил, получаем неравенство:

Кроме того, по условию,

Так как правильный ответ добавляет 7 очков, а неправильный убавляет 9, и в конечном итоге ученик набрал 56 очков, получаем уравнение:

Это уравнение надо решить в целых числах.

Так как 9 на 7 не делится, должен делиться на 7.

Пусть , тогда .

В этом случае - все условия выполняются.

Задача №10056.

Прямоугольник разбит на четыре маленьких прямоугольника двумя прямолинейными разрезами. Площади трех из них, начиная с левого верхнего и далее по часовой стрелке равны 15, 18, 24. Найдите площадь четвертого прямоугольника.


Площадь прямоугольника равна произведению его сторон.

Желтый и голубой прямоугольники имеют общую сторону, поэтому отношение площадей этих прямоугольников равно отношению длин других сторон (не равных между собой).

Белый и зеленый прямоугольники также имеют имеют общую сторону, поэтому отношение их площадей равно отношению других сторон (не равных между собой), то есть тому же отношению:

По свойству пропорции получим

Отсюда .

Задача №10071.

Прямоугольник разбит на четыре маленьких прямоугольника двумя прямолинейными разрезами. Периметры трех из них, начиная с левого верхнего и далее почасовой стрелке равны 17, 12, 13. Найдите периметр четвертого прямоугольника.


Периметр прямоугольника равен сумме длин всех его сторон.

Обозначим стороны прямоугольников как указано на рисунке и выразим через указанные переменные периметры прямоугольников. Получим:

Теперь нам нужно найти, чему равно значение выражения .

Вычтем из третьего уравнения второе и прибавим третье. Получим:

Упростим правую и левую части, получим:

Итак, .

Ответ: 18.

Задача №10086.

В таблице три столбца и несколько строк. В каждую клетку таблицы поставили по натуральному числу так, что сумма всех чисел в первом столбце равна 72, во втором – 81, в третьем – 91, а сумма чисел в каждой строке больше 13, но меньше 16. Сколько всего строк в таблице?

Найдем сумму всех чисел в таблице: .

Пусть число строк в таблице равно .

По условию задачи сумма чисел в каждой строке больше 13, но меньше 16 .

Так как сумма чисел - натуральное число, этому двойному неравенству удовлетворяют только два натуральных числа: 14 и 15.

Если предположить, что сумма чисел в каждой строке равна 14, то тогда сумма всех чисел в таблице равна , и эта сумма удовлетворяет неравенству .

Если предположить, что сумма чисел в каждой строке равна 15, то тогда сумма всех чисел в таблице равна , и это число удовлетворяет неравенству .

Итак, натуральное число должно удовлетворять системе неравенств:

Единственное натуральное , удовлетворяющее этой системе - это

Ответ: 17.

Про натуральные числа А, В и С известно, что каждое из них больше 4 но меньше 8. Загадали натуральное число, затем его умножили на А потом прибавили к полученному произведению В и вычли С. Получилось 165. Какое число было загадано?

Натуральные числа А, В и С могут быть равны числам 5, 6 или 7.

Пусть неизвестное натуральное число равно .

Получим: ;

Рассмотрим различные варианты.

Пусть А=5. Тогда B=6 и С=7, или B=7 и С=6, или B=7 и С=7, или B=6 и С=6.

Проверим: ; (1)

165 делится на 5.

Разность между числами В и С либо равна , либо равна 0, если эти числа равны. Если разность равна , то равенство (1) невозможно. Следовательно, разность равна 0 и

Пусть А=6. Тогда B=5 и С=7, или B=7 и С=5, или B=7 и С=7, или B=5 и С=5.

Проверим: ; (2)

Разность между числами В и С либо равна , либо равна 0, если эти числа равны. Если разность равна или 0 то равенство (2) невозможно, так как - четное число, а сумма (165 + четное число) - не может быть четным числом.

Пусть А=7. Тогда B=5 и С=6, или B=6 и С=5, или B=6 и С=6, или B=5 и С=5.

Проверим: ; (3)

Разность между числами В и С либо равна , либо равна 0, если эти числа равны. Число 165 при делении на 7 дает в остатке 4. Следовательно, также не делится на 7, и равенство (3) невозможно.

Ответ: 33

Из книги выпало несколько идущих подряд листов. Номер последней страницы перед выпавшими листами - 352, номер первой страницы после выпавших листов записывается теми же цифрами, но в другом порядке. Сколько листов выпало?

Очевидно, что номер первой страницы после выпавших листов больше чем 352, значит это может быть либо 532, либо 523.

Каждый выпавший лист содержит 2 страницы. Следовательно выпало четное число страниц. 352 - четное число. Если мы к четному числу прибавим четное, то получим четное число. Следовательно, номер последней выпавшей страницы - четное число, и номер первой страницы после выпавших листов должен быть нечетным, то есть 523. Следовательно, номер последней выпавшей страницы 522. Тогда выпало листов.

Ответ: 85

Маша и Медведь съели 160 печений и банку варенья, начав и закончив одновременно. Сначала Маша ела варенье, а Медведь - печенье, но в какой-то момент они поменялись. Медведь и то, и другое ест в три раза быстрее Маши. Сколько печений съел Медведь, если варенье они съели поровну?

Если Маша и Медведь съели варенье поровну, а медведь в единицу времени съедал втрое больше варенья, значит он ел варенье втрое меньшее время, чем Маша. Другим словами, Маша ела варенье втрое дольше, чем Медведь. Но пока Маша ела варенье, медведь ел печенье. Следовательно, медведь ел печенье втрое дольше, чем Маша. Но Медведь, к тому же, в единицу времени съедал втрое больше печенья, чем Маша, следовательно, в итоге он съел в 9 раз больше печенья, чем Маша.

Теперь несложно составить уравнение. Пусть Маша съела печений, тогда Медведь съел печений. Вместе они съели печений. получаем уравнение:

Ответ: 144

На прилавке цветочного магазина стоят 3 вазы с розами: оранжевая, белая и синяя. Слева от оранжевой вазы 15 роз, справа от синей вазы 12 роз. Всего в вазах 22 розы. сколько роз в оранжевой вазе?

Так как 15+12=27, и 27>22, следовательно, количество цветов одной вазе посчитали дважды. И это белая ваза, так как это должная быть ваза, которая стоит справа от синей и слева от оранжевой. Значит, вазы стоят в таком порядке:

Отсюда получаем систему:

Вычтя из третьего уравнения первое, получим О= 7.

Ответ: 7

Десять столбов соединены между собой проводами так, что от каждого столба отходит ровно 8 проводов. сколько всего проводов протянуто между этими десятью столбами?

Решение

Смоделируем ситуацию. Пусть у нас есть два столба, и они соединены между собой проводами так, что от каждого столба отходит ровно 1 провод. Тогда получается, что от столбов отходит 2 провода. Но мы имеем такую ситуацию:


То есть при том, что от столбов отходит 2 провода, протянут между столбами всего один провод. Значит, число протянутых проводов в два раза меньше, чем число отходящих.

Получаем: - число отходящих проводов.

Число протянутых проводов.

Ответ: 40

Из десяти стран семь подписали договор о дружбе ровно с тремя другими странами, а каждая из оставшихся трёх - ровно с семью. Сколько всего было подписано договоров?

Эта задача аналогична предыдущей: две страны подписывают один общий договор. На каждом договоре стоит две подписи. То есть число подписанных договоров вдвое меньше, чем число подписей.

Найдем число подписей:

Найдем число подписанных договоров:

Ответ: 21

Три луча, выходящие из одной точки, разбивают плоскость на три разных угла, измеряемых целым числом градусов. Наибольший угол в 3 раза больше наименьшего. Сколько значений может принимать величина среднего угла?

Пусть наименьший угол равен , тогда наибольший угол равен . Так как сумма всех углов равна , величина среднего угла равна .


Средний угол должен больше наименьшего и меньше наибольшего угла.

Получим систему неравенств:

Следовательно, принимает значения в диапазоне от 52 до 71 градуса, то есть всего возможных значений.

Ответ: 20

Миша, Коля и Леша играют в настольный теннис: игрок, проигравший партию, уступает место игроку, не участвовавшему в ней. В итоге оказалось, что Миша сыграл 12 партий, а Коля - 25. Сколько партий сыграл Леша?

Решение

Следует пояснить, как устроен турнир: турнир состоит из фиксированного числа партий; проигравший в данной партии игрок уступает место игроку, который не участвовал в данной партии. По итогам следующей партии игрок, который не принимал в ней участие, заступает на место проигравшего. Следовательно, каждый игрок принимает участие хотя бы в одной из двух последовательных партий.

Найдем, сколько всего было партий.

Так как Коля сыграл 25 партий, следовательно, в турнире было проведено не меньше 25 партий.

Миша сыграл 12 партий. Так как он точно принимал участие в каждой второй партии, следовательно, было проведено не больше, чем партий. То есть турнир состоял из 25 партий.

Если Миша сыграл 12 партий, то Леша сыграл оставшиеся 13.

Ответ: 13

В конце четверти Петя выписал подряд все свои отметки по одному из предметов, их оказалось 5, и поставил между некоторыми из них знаки умножения. Произведение получившихся чисел оказалось равным 3495 . Какая отметка выходит у Пети в четверти по этому предмету, если учитель ставит только отметки 2, 3, 4 или 5 и итоговая отметка в четверти является средним арифметическим всех текущих отметок, округленным по правилам округления? (Например, 3,2 округляется до 3; 4,5 - до 5; 2,8 - до 3)

Разложим 3495 на простые множители. Последняя цифра числа 5, следовательно, число делится на 5; сумма цифр делится на 3, следовательно число делится на 3.

Получили, что

Следовательно, оценки Пети 3, 5, 2, 3, 3. Найдем среднее арифметическое:

Ответ: 3

Среднее арифметическое 6 различных натуральных чисел равно 8. На сколько нужно увеличить наибольшее из этих чисел, чтобы их среднее арифметическое стало на 1 больше?

Среднее арифметическое равно сумме всех чисел, деленной на их количество. Пусть сумма всех чисел равна . По условию задачи , следовательно .

Среднее арифметическое стало на 1 больше, то есть стало равно 9. Если одно из чисел увеличили на , то и сумма увеличилась на и стала равна .

Количество чисел не изменилось и равно 6.

Получаем равенство:

Среднее общее образование

Линия УМК Г. К. Муравина. Алгебра и начала математического анализа (10-11) (углуб.)

Линия УМК Мерзляка. Алгебра и начала анализа (10-11) (У)

Математика

Подготовка к ЕГЭ по математике (профильный уровень): задания, решения и объяснения

Разбираем задания и решаем примеры с учителем

Экзаменационная работа профильного уровня длится 3 часа 55 минут (235 минут).

Минимальный порог - 27 баллов.

Экзаменационная работа состоит из двух частей, которые различаются по содержанию, сложности и числу заданий.

Определяющим признаком каждой части работы является форма заданий:

  • часть 1 содержит 8 заданий (задания 1-8) с кратким ответом в виде целого числа или конечной десятичной дроби;
  • часть 2 содержит 4 задания (задания 9-12) с кратким ответом в виде целого числа или конечной десятичной дроби и 7 заданий (задания 13–19) с развернутым ответом (полная запись решения с обоснованием выполненных действий).

Панова Светлана Анатольевна , учитель математики высшей категории школы, стаж работы 20 лет:

«Для того чтобы получить школьный аттестат, выпускнику необходимо сдать два обязательных экзамена в форме ЕГЭ, один из которых математика. В соответствии с Концепцией развития математического образования в Российской Федерации ЕГЭ по математике разделен на два уровня: базовый и профильный. Сегодня мы рассмотрим варианты профильного уровня».

Задание № 1 - проверяет у участников ЕГЭ умение применять навыки, полученные в курсе 5 - 9 классов по элементарной математике, в практической деятельности. Участник должен владеть вычислительными навыками, уметь работать с рациональными числами, уметь округлять десятичные дроби, уметь переводить одни единицы измерения в другие.

Пример 1. В квартире, где проживает Петр, установили прибор учета расхода холодной воды (счетчик). Первого мая счетчик показывал расход 172 куб. м воды, а первого июня - 177 куб. м. Какую сумму должен заплатить Петр за холодную воду за май, если цена 1 куб. м холодной воды составляет 34 руб 17 коп? Ответ дайте в рублях.

Решение:

1) Найдем количество потраченной воды за месяц:

177 - 172 = 5 (куб м)

2) Найдем сколько денег заплатят за потраченную воду:

34,17 · 5 = 170,85 (руб)

Ответ: 170,85.


Задание № 2 -является одним из простейших заданий экзамена. С ней успешно справляется большинство выпускников, что свидетельствует о владении определением понятия функции. Тип задания № 2 по кодификатору требований - это задание на использования приобретённых знаний и умений в практической деятельности и повседневной жизни. Задание № 2 состоит из описания с помощью функций различных реальных зависимостей между величинами и интерпретация их графиков. Задание № 2 проверяет умение извлекать информацию, представленную в таблицах, на диаграммах, графиках. Выпускникам нужно уметь определять значение функции по значению аргумента при различных способах задания функции и описывать поведение и свойства функции по её графику. Также необходимо уметь находить по графику функции наибольшее или наименьшее значение и строить графики изученных функций. Допускаемые ошибки носят случайный характер в чтении условия задачи, чтении диаграммы.

#ADVERTISING_INSERT#

Пример 2. На рисунке показано изменение биржевой стоимости одной акции добывающей компании в первой половине апреля 2017 года. 7 апреля бизнесмен приобрёл 1000 акций этой компании. 10 апреля он продал три четверти купленных акций, а 13 апреля продал все оставшиеся. Сколько потерял бизнесмен в результате этих операций?


Решение:

2) 1000 · 3/4 = 750 (акций) - составляют 3/4 от всех купленных акций.

6) 247500 + 77500 = 325000 (руб) - бизнесмен получил после продажи 1000 акций.

7) 340000 – 325000 = 15000 (руб) - потерял бизнесмен в результате всех операций.

Ответ: 15000.

Задание № 3 - является заданием базового уровня первой части, проверяет умения выполнять действия с геометрическими фигурами по содержанию курса «Планиметрия». В задании 3 проверяется умение вычислять площадь фигуры на клетчатой бумаге, умение вычислять градусные меры углов, вычислять периметры и т.п.

Пример 3. Найдите площадь прямоугольника, изображенного на клетчатой бумаге с размером клетки 1 см на 1 см (см. рис.). Ответ дайте в квадратных сантиметрах.

Решение: Для вычисления площади данной фигуры можно воспользоваться формулой Пика:

Для вычисления площади данного прямоугольника воспользуемся формулой Пика:

S = В +

Г
2
где В = 10, Г = 6, поэтому

S = 18 +

6
2
Ответ: 20.

Читайте также: ЕГЭ по физике: решение задач о колебаниях

Задание № 4 - задача курса «Теория вероятностей и статистика». Проверяется умение вычислять вероятность события в простейшей ситуации.

Пример 4. На окружности отмечены 5 красных и 1 синяя точка. Определите, каких многоугольников больше: тех, у которых все вершины красные, или тех, у которых одна из вершин синяя. В ответе укажите, на сколько одних больше, чем других.

Решение: 1) Воспользуемся формулой числа сочетаний из n элементов по k :

у которых все вершины красные.

3) Один пятиугольник, у которого все вершины красные.

4) 10 + 5 + 1 = 16 многоугольников, у которых все вершины красные.

у которых вершины красные или с одной синей вершиной.

у которых вершины красные или с одной синей вершиной.

8) Один шестиуголник, у которого вершины красные с одной синей вершиной.

9) 20 + 15 + 6 + 1 = 42 многоуголника, у которых все вершины красные или с одной синей вершиной.

10) 42 – 16 = 26 многоугольников, в которых используется синяя точка.

11) 26 – 16 = 10 многоугольников – на сколько многоугольников, у которых одна из вершин - синяя точка, больше, чем многоугольников, у которых все вершины только красные.

Ответ: 10.

Задание № 5 - базового уровня первой части проверяет умения решать простейшие уравнения (иррациональные, показательные, тригонометрические, логарифмические).

Пример 5. Решите уравнение 2 3 + x = 0,4 · 5 3 + x .

Решение. Разделим обе части данного уравнения на 5 3 + х ≠ 0, получим

2 3 + x = 0,4 или 2 3 + х = 2 ,
5 3 + х 5 5

откуда следует, что 3 + x = 1, x = –2.

Ответ: –2.

Задание № 6 по планиметрии на нахождение геометрических величин (длин, углов, площадей), моделирование реальных ситуаций на языке геометрии. Исследование построенных моделей с использованием геометрических понятий и теорем. Источником трудностей является, как правило, незнание или неверное применение необходимых теорем планиметрии.

Площадь треугольника ABC равна 129. DE – средняя линия, параллельная стороне AB . Найдите площадь трапеции ABED .


Решение. Треугольник CDE подобен треугольнику CAB по двум углам, так как угол при вершине C общий, угол СDE равен углу CAB как соответственные углы при DE || AB секущей AC . Так как DE – средняя линия треугольника по условию, то по свойству средней линии | DE = (1/2)AB . Значит, коэффициент подобия равен 0,5. Площади подобных фигур относятся как квадрат коэффициента подобия, поэтому

Следовательно, S ABED = S ΔABC S ΔCDE = 129 – 32,25 = 96,75.

Задание № 7 - проверяет применение производной к исследованию функции. Для успешного выполнения необходимо содержательное, не формальное владение понятием производной.

Пример 7. К графику функции y = f (x ) в точке с абсциссой x 0 проведена касательная, которая перпендикулярна прямой, проходящей через точки (4; 3) и (3; –1) этого графика. Найдите f ′(x 0).

Решение. 1) Воспользуемся уравнением прямой, проходящей через две заданные точки и найдём уравнение прямой, проходящей через точки (4; 3) и (3; –1).

(y y 1)(x 2 – x 1) = (x x 1)(y 2 – y 1)

(y – 3)(3 – 4) = (x – 4)(–1 – 3)

(y – 3)(–1) = (x – 4)(–4)

y + 3 = –4x + 16| · (–1)

y – 3 = 4x – 16

y = 4x – 13, где k 1 = 4.

2) Найдём угловой коэффициент касательной k 2 , которая перпендикулярна прямой y = 4x – 13, где k 1 = 4, по формуле:

3) Угловой коэффициент касательной – производная функции в точке касания. Значит, f ′(x 0) = k 2 = –0,25.

Ответ: –0,25.

Задание № 8 - проверяет у участников экзамена знания по элементарной стереометрии, умение применять формулы нахождения площадей поверхностей и объемов фигур, двугранных углов, сравнивать объемы подобных фигур, уметь выполнять действия с геометрическими фигурами, координатами и векторами и т.п.

Объём куба, описанного около сферы, равен 216. Найдите радиус сферы.


Решение. 1) V куба = a 3 (где а – длина ребра куба), поэтому

а 3 = 216

а = 3 √216

2) Так как сфера вписана в куб, значит, длина диаметра сферы равна длине ребра куба, поэтому d = a , d = 6, d = 2R , R = 6: 2 = 3.

Задание № 9 - требует от выпускника навыков преобразования и упрощения алгебраических выражений. Задание № 9 повышенного уровня сложности с кратким ответом. Задания из раздела «Вычисления и преобразования» в ЕГЭ подразделяются на несколько видов:

    преобразования числовых рациональных выражений;

    преобразования алгебраических выражений и дробей;

    преобразования числовых/буквенных иррациональных выражений;

    действия со степенями;

    преобразование логарифмических выражений;

  1. преобразования числовых/буквенных тригонометрических выражений.

Пример 9. Вычислите tgα, если известно, что cos2α = 0,6 и

< α < π.
4

Решение. 1) Воспользуемся формулой двойного аргумента: cos2α = 2 cos 2 α – 1 и найдём

tg 2 α = 1 – 1 = 1 – 1 = 10 – 1 = 5 – 1 = 1 1 – 1 = 1 = 0,25.
cos 2 α 0,8 8 4 4 4

Значит, tg 2 α = ± 0,5.

3) По условию

< α < π,
4

значит, α – угол II четверти и tgα < 0, поэтому tgα = –0,5.

Ответ: –0,5.

#ADVERTISING_INSERT# Задание № 10 - проверяет у учащихся умение использовать приобретенные раннее знания и умения в практической деятельности и повседневной жизни. Можно сказать, что это задачи по физике, а не по математике, но все необходимые формулы и величины даны в условии. Задачи сводятся к решению линейного или квадратного уравнения, либо линейного или квадратного неравенства. Поэтому необходимо уметь решать такие уравнения и неравенства, и определять ответ. Ответ должен получиться в виде целого числа или конечной десятичной дроби.

Два тела массой m = 2 кг каждое, движутся с одинаковой скоростью v = 10 м/с под углом 2α друг к другу. Энергия (в джоулях), выделяющаяся при их абсолютно неупругом соударении определяется выражением Q = mv 2 sin 2 α. Под каким наименьшим углом 2α (в градусах) должны двигаться тела, чтобы в результате соударения выделилось не менее 50 джоулей?
Решение. Для решения задачи нам необходимо решить неравенство Q ≥ 50, на интервале 2α ∈ (0°; 180°).

mv 2 sin 2 α ≥ 50

2· 10 2 sin 2 α ≥ 50

200 · sin 2 α ≥ 50

Так как α ∈ (0°; 90°), то будем решать только

Изобразим решение неравенства графически:


Так как по условию α ∈ (0°; 90°), значит 30° ≤ α < 90°. Получили, что наименьший угол α равен 30°, тогда наименьший угол 2α = 60°.

Задание № 11 - является типовым, но оказывается непростым для учащихся. Главным источником затруднений является построение математической модели (составление уравнения). Задание № 11 проверяет умение решать текстовые задачи.

Пример 11. На весенних каникулах 11-классник Вася должен был решить 560 тренировочных задач для подготовки к ЕГЭ. 18 марта в последний учебный день Вася решил 5 задач. Далее ежедневно он решал на одно и то же количество задач больше по сравнению с предыдущим днём. Определите, сколько задач Вася решил 2 апреля в последний день каникул.

Решение: Обозначим a 1 = 5 – количество задач, которые Вася решил 18 марта, d – ежедневное количество задач, решаемых Васей, n = 16 – количество дней с 18 марта по 2 апреля включительно, S 16 = 560 – общее количество задач, a 16 – количество задач, которые Вася решил 2 апреля. Зная, что ежедневно Вася решал на одно и то же количество задач больше по сравнению с предыдущим днём, то можно использовать формулы нахождения суммы арифметической прогрессии:

560 = (5 + a 16) · 8,

5 + a 16 = 560: 8,

5 + a 16 = 70,

a 16 = 70 – 5

a 16 = 65.

Ответ: 65.

Задание № 12 - проверяют у учащихся умение выполнять действия с функциями, уметь применять производную к исследованию функции.

Найти точку максимума функции y = 10ln(x + 9) – 10x + 1.

Решение: 1) Найдем область определения функции: x + 9 > 0, x > –9, то есть x ∈ (–9; ∞).

2) Найдем производную функции:

4) Найденная точка принадлежит промежутку (–9; ∞). Определим знаки производной функции и изобразим на рисунке поведение функции:


Искомая точка максимума x = –8.

Скачать бесплатно рабочую программу по математике к линии УМК Г.К. Муравина, К.С. Муравина, О.В. Муравиной 10-11 Скачать бесплатно методические пособия по алгебре

Задание № 13 -повышенного уровня сложности с развернутым ответом, проверяющее умение решать уравнения, наиболее успешно решаемое среди заданий с развернутым ответом повышенного уровня сложности.

а) Решите уравнение 2log 3 2 (2cosx ) – 5log 3 (2cosx ) + 2 = 0

б) Найдите все корни этого уравнения, принадлежащие отрезку .

Решение: а) Пусть log 3 (2cosx ) = t , тогда 2t 2 – 5t + 2 = 0,


log 3 (2cosx ) = 2
2cosx = 9
cosx = 4,5 ⇔ т.к. |cosx | ≤ 1,
log 3 (2cosx ) = 1 2cosx = √3 cosx = √3
2 2
то cosx = √3
2

x = π + 2πk
6
x = – π + 2πk , k Z
6

б) Найдём корни, лежащие на отрезке .


Из рисунка видно, что заданному отрезку принадлежат корни

11π и 13π .
6 6
Ответ: а) π + 2πk ; – π + 2πk , k Z ; б) 11π ; 13π .
6 6 6 6
Задание № 14 -повышенного уровня относится к заданиям второй части с развернутым ответом. Задание проверяет умения выполнять действия с геометрическими фигурами. Задание содержит два пункта. В первом пункте задание нужно доказать, а во втором пункте вычислить.

Диаметр окружности основания цилиндра равен 20, образующая цилиндра равна 28. Плоскость пересекает его основания по хордам длины 12 и 16. Расстояние между хордами равно 2√197.

а) Докажите, что центры оснований цилиндра лежат по одну сторону от этой плоскости.

б) Найдите угол между этой плоскостью и плоскостью основания цилиндра.

Решение: а) Хорда длиной 12 находится на расстоянии = 8 от центра окружности основания, а хорда длиной 16, аналогично, – на расстоянии 6. Поэтому расстояние между их проекциями на плоскость, параллельную основаниям цилиндров, составляет либо 8 + 6 = 14, либо 8 − 6 = 2.

Тогда расстояние между хордами составляет либо

= = √980 = = 2√245

= = √788 = = 2√197.

По условию реализовался второй случай, в нем проекции хорд лежат по одну сторону от оси цилиндра. Значит, ось не пересекает данную плоскость в пределах цилиндра, то есть основания лежат по одну сторону от нее. Что требовалось доказать.

б) Обозначим центры оснований за О 1 и О 2 . Проведем из центра основания с хордой длины 12 серединный перпендикуляр к этой хорде (он имеет длину 8, как уже отмечалось) и из центра другого основания - к другой хорде. Они лежат в одной плоскости β, перпендикулярной этим хордам. Назовем середину меньшей хорды B, большей A и проекцию A на второе основание - H (H ∈ β). Тогда AB,AH ∈ β и значит, AB,AH перпендикулярны хорде, то есть прямой пересечения основания с данной плоскостью.

Значит, искомый угол равен

∠ABH = arctg AH = arctg 28 = arctg14.
BH 8 – 6

Задание № 15 - повышенного уровня сложности с развернутым ответом, проверяет умение решать неравенства, наиболее успешно решаемое среди заданий с развернутым ответом повышенного уровня сложности.

Пример 15. Решите неравенство |x 2 – 3x | · log 2 (x + 1) ≤ 3x x 2 .

Решение: Областью определения данного неравенства является интервал (–1; +∞). Рассмотри отдельно три случая:

1) Пусть x 2 – 3x = 0, т.е. х = 0 или х = 3. В этом случае данное неравенство превращается в верное, следовательно, эти значения входят в решение.

2) Пусть теперь x 2 – 3x > 0, т.е. x ∈ (–1; 0) ∪ (3; +∞). При этом данное неравенство можно переписать в виде (x 2 – 3x ) · log 2 (x + 1) ≤ 3x x 2 и разделить на положительное выражение x 2 – 3x . Получим log 2 (x + 1) ≤ –1, x + 1 ≤ 2 –1 , x ≤ 0,5 –1 или x ≤ –0,5. Учитывая область определения, имеем x ∈ (–1; –0,5].

3) Наконец, рассмотрим x 2 – 3x < 0, при этом x ∈ (0; 3). При этом исходное неравенство перепишется в виде (3x x 2) · log 2 (x + 1) ≤ 3x x 2 . После деления на положительное выражение 3x x 2 , получим log 2 (x + 1) ≤ 1, x + 1 ≤ 2, x ≤ 1. Учитывая область, имеем x ∈ (0; 1].

Объединяя полученные решения, получаем x ∈ (–1; –0.5] ∪ ∪ {3}.

Ответ: (–1; –0.5] ∪ ∪ {3}.

Задание № 16 - повышенного уровня относится к заданиям второй части с развернутым ответом. Задание проверяет умения выполнять действия с геометрическими фигурами, координатами и векторами. Задание содержит два пункта. В первом пункте задание нужно доказать, а во втором пункте вычислить.

В равнобедренном треугольнике ABC с углом 120° при вершине A проведена биссектриса BD. В треугольник ABC вписан прямоугольник DEFH так, что сторона FH лежит на отрезке BC, а вершина E – на отрезке AB. а) Докажите, что FH = 2DH. б) Найдите площадь прямоугольника DEFH, если AB = 4.

Решение: а)


1) ΔBEF – прямоугольный, EF⊥BC, ∠B = (180° – 120°) : 2 = 30°, тогда EF = BE по свойству катета, лежащего против угла 30°.

2) Пусть EF = DH = x , тогда BE = 2x , BF = x √3 по теореме Пифагора.

3) Так как ΔABC равнобедренный, значит, ∠B = ∠C = 30˚.

BD – биссектриса ∠B, значит ∠ABD = ∠DBC = 15˚.

4) Рассмотрим ΔDBH – прямоугольный, т.к. DH⊥BC.

2x = 4 – 2x
2x (√3 + 1) 4
1 = 2 – x
√3 + 1 2

√3 – 1 = 2 – x

x = 3 – √3

EF = 3 – √3

2) S DEFH = ED · EF = (3 – √3 ) · 2(3 – √3 )

S DEFH = 24 – 12√3.

Ответ: 24 – 12√3.


Задание № 17 - задание с развернутым ответом, это задание проверяет применение знаний и умений в практической деятельности и повседневной жизни, умение строить и исследовать математические модели. Это задание - текстовая задача с экономическим содержанием.

Пример 17. Вклад в размере 20 млн рублей планируется открыть на четыре года. В конце каждого года банк увеличивает вклад на 10% по сравнению с его размером в начале года. Кроме того, в начале третьего и четвёртого годов вкладчик ежегодно пополняет вклад на х млн. рублей, где х - целое число. Найдите наибольшее значение х , при котором банк за четыре года начислит на вклад меньше 17 млн рублей.

Решение: В конце первого года вклад составит 20 + 20 · 0,1 = 22 млн рублей, а в конце второго – 22 + 22 · 0,1 = 24,2 млн рублей. В начале третьего года вклад (в млн рублей) составит (24,2 + х ), а в конце - (24,2 + х) + (24,2 + х) · 0,1 = (26,62 + 1,1х ). В начале четвёртого года вклад составит (26,62 + 2,1х) , а в конце - (26,62 + 2,1х ) + (26,62 + 2,1х ) · 0,1 = (29,282 + 2,31х ). По условию, нужно найти наибольшее целое х, для которого выполнено неравенство

(29,282 + 2,31x ) – 20 – 2x < 17

29,282 + 2,31x – 20 – 2x < 17

0,31x < 17 + 20 – 29,282

0,31x < 7,718

x < 7718
310
x < 3859
155
x < 24 139
155

Наибольшее целое решение этого неравенства - число 24.

Ответ: 24.


Задание № 18 - задание повышенного уровня сложности с развернутым ответом. Это задание предназначено для конкурсного отбора в вузы с повышенными требованиями к математической подготовке абитуриентов. Задание высокого уровня сложности - это задание не на применение одного метода решения, а на комбинацию различных методов. Для успешного выполнения задания 18 необходим, кроме прочных математических знаний, также высокий уровень математической культуры.

При каких a система неравенств

x 2 + y 2 ≤ 2ay a 2 + 1
y + a ≤ |x | – a

имеет ровно два решения?

Решение: Данную систему можно переписать в виде

x 2 + (y a ) 2 ≤ 1
y ≤ |x | – a

Если нарисовать на плоскости множество решений первого неравенства, получится внутренность круга (с границей) радиуса 1 с центром в точке (0, а ). Множество решений второго неравенства – часть плоскости, лежащая под графиком функции y = | x | – a , причём последний есть график функции
y = | x | , сдвинутый вниз на а . Решение данной системы есть пересечение множеств решений каждого из неравенств.

Следовательно, два решения данная система будет иметь лишь в случае, изображённом на рис. 1.


Точки касания круга с прямыми и будут двумя решениями системы. Каждая из прямых наклонена к осям под углом 45°. Значит, треугольник PQR – прямоугольный равнобедренный. Точка Q имеет координаты (0, а ), а точка R – координаты (0, –а ). Кроме того, отрезки PR и PQ равны радиусу окружности, равному 1. Значит,

Qr = 2a = √2, a = √2 .
2
Ответ: a = √2 .
2


Задание № 19 - задание повышенного уровня сложности с развернутым ответом. Это задание предназначено для конкурсного отбора в вузы с повышенными требованиями к математической подготовке абитуриентов. Задание высокого уровня сложности - это задание не на применение одного метода решения, а на комбинацию различных методов. Для успешного выполнения задания 19 необходимо уметь осуществлять поиск решения, выбирая различные подходы из числа известных, модифицируя изученные методы.

Пусть Sn сумма п членов арифметической прогрессии (а п ). Известно, что S n + 1 = 2n 2 – 21n – 23.

а) Укажите формулу п -го члена этой прогрессии.

б) Найдите наименьшую по модулю сумму S n .

в) Найдите наименьшее п , при котором S n будет квадратом целого числа.

Решение : а) Очевидно, что a n = S n S n – 1 . Используя данную формулу, получаем:

S n = S (n – 1) + 1 = 2(n – 1) 2 – 21(n – 1) – 23 = 2n 2 – 25n ,

S n – 1 = S (n – 2) + 1 = 2(n – 1) 2 – 21(n – 2) – 23 = 2n 2 – 25n + 27

значит, a n = 2n 2 – 25n – (2n 2 – 29n + 27) = 4n – 27.

Б) Так как S n = 2n 2 – 25n , то рассмотрим функцию S (x ) = | 2x 2 – 25x| . Ее график можно увидеть на рисунке.


Очевидно, что наименьшее значение достигается в целочисленных точках, расположенных наиболее близко к нулям функции. Очевидно, что это точки х = 1, х = 12 и х = 13. Поскольку, S (1) = |S 1 | = |2 – 25| = 23, S (12) = |S 12 | = |2 · 144 – 25 · 12| = 12, S (13) = |S 13 | = |2 · 169 – 25 · 13| = 13, то наименьшее значение равно 12.

в) Из предыдущего пункта вытекает, что Sn положительно, начиная с n = 13. Так как S n = 2n 2 – 25n = n (2n – 25), то очевидный случай, когда данное выражение является полным квадратом, реализуется при n = 2n – 25, то есть при п = 25.

Осталось проверить значения с 13 до 25:

S 13 = 13 · 1, S 14 = 14 · 3, S 15 = 15 · 5, S 16 = 16 · 7, S 17 = 17 · 9, S 18 = 18 · 11, S 19 = 19 · 13, S 20 = 20 · 13, S 21 = 21 · 17, S 22 = 22 · 19, S 23 = 23 · 21, S 24 = 24 · 23.

Получается, что при меньших значениях п полный квадрат не достигается.

Ответ: а) a n = 4n – 27; б) 12; в) 25.

________________

*С мая 2017 года объединенная издательская группа «ДРОФА-ВЕНТАНА» входит в корпорацию «Российский учебник». В корпорацию также вошли издательство «Астрель» и цифровая образовательная платформа «LECTA». Генеральным директором назначен Александр Брычкин, выпускник Финансовой академии при Правительстве РФ, кандидат экономических наук, руководитель инновационных проектов издательства «ДРОФА» в сфере цифрового образования (электронные формы учебников, «Российская электронная школа», цифровая образовательная платформа LECTA). До прихода в издательство «ДРОФА» занимал позицию вице-президента по стратегическому развитию и инвестициям издательского холдинга «ЭКСМО-АСТ». Сегодня издательская корпорация «Российский учебник» обладает самым крупным портфелем учебников, включенных в Федеральный перечень - 485 наименований (примерно 40%, без учета учебников для коррекционной школы). Издательствам корпорации принадлежат наиболее востребованные российскими школами комплекты учебников по физике, черчению, биологии, химии, технологии, географии, астрономии - областям знаний, которые нужны для развития производственного потенциала страны. В портфель корпорации входят учебники и учебные пособия для начальной школы, удостоенные Премии Президента в области образования. Это учебники и пособия по предметным областям, которые необходимы для развития научно-технического и производственного потенциала России.